AUD MC

¡Supera tus tareas y exámenes ahora con Quizwiz!

To obtain audit evidence about control risk, an auditor selects tests from a variety of techniques including: a. Inquiry. b. Analytical procedures. c. Calculation. d. Confirmation.

d. Confirmation. Tests of controls include such procedures as inspecting documentation, inquiry, observation, and reperformance. Note that inquiry alone is not sufficient as the auditor should use a combination of procedures.

The most likely explanation why the auditor's examination cannot reasonably be expected to bring all acts of noncompliance with laws and regulations by the client to the auditor's attention is that: A. Acts of noncompliance with laws and regulations by clients often relate to operating aspects rather than accounting aspects B. Acts of noncompliance with laws and regulations are perpetrated by management override in internal accounting controls C. Acts of noncompliance with laws and regulations may be perpetrated by the only person in the client's organization with access to both assets and the accounting records D. The client's system of internal control may be so strong that the auditor performs only minimal substantive testing

A. Acts of noncompliance with laws and regulations by clients often relate to operating aspects rather than accounting aspects The audit should be designed to identify material misstatements due to acts of noncompliance with laws and regulations, but acts of noncompliance with laws and regulations that relate to operating aspects rather than accounting aspects may not directly affect the financial statements, and therefore they may be less likely to be discovered by the auditor.

which of the following activities is not an element of a CPA Firm's quality-control system to be considered in established quality-control policies and procedures? A. Assessing a client's ability to establish effective internal controls B. Deciding whether to accept or continue a client relationship C. Selecting personnel for advancement who have the necessary qualifications D. Monitoring the effectiveness of professional development activities

A. Assessing a client's ability to establish effective internal controls

Which of the following statements is correct concerning an auditor's assessment of control risk? A. Assessing control risk may be performed concurrently during an audit with obtaining an understanding of the entity's internal control B. Evidence about the operation of controls in prior audits may not be considered during the current year's assessment of control risk C. The basis for an auditor's conclusions about the assessed level of control risk need not be documented unless the auditor's risk assessment is based on the effective operation of controls D. The lower the assessed level of control risk, the less assurance the evidence must provide that the controls are operating effectively.

A. Assessing control risk may be performed concurrently during an audit with obtaining an understanding of the entity's internal control Some risk assessment procedures performed to obtain an understanding of internal control may provide evidence about operating effectiveness, even if they were not intended for that purpose. This is possible because the procedures performed to achieve one objective may also provide evidence regarding the other. Choice "4" is incorrect. The lower the assessed level of control risk (that is, the greater the reliance placed on the control), the more assurance the auditor must gather that the controls are operating effectively.

Which of the following is not true regarding audit documentation for a specific audit? A. Audit documentation should demonstrate compliance with quality control standards B. Audit documentation should demonstrate compliance with the standards of fieldwork C. Audit documentation should indicate which members of the audit team performed and reviewed the audit work D. Audit documentation should be sufficient to enable members of the audit team with supervisory responsibilities to understand the nature, timing, extent and results of auditing procedures performed

A. Audit documentation should demonstrate compliance with quality control standards Quality control standards relate to the conduct of a firm's audit practice as a whole, and compliance with such standards would not be demonstrated by audit documentation for one specific audit engagement.

In using the work of a specialist, an auditor referred to the specialist's findings in the auditor's report. This would be an appropriate reporting practice if the : A. Auditor, as a result of the specialist's findings, adds an explanatory paragraph in a modified opinion emphasizing a matter regarding the financial statements B. Auditor understands the form and content of the specialist's findings in relation to the representations in the financial statements C. Auditor, as a result of the specialist's findings, decides to indicate a division of responsibility with the specialist D. Client is not familiar with the professional certification, personal reputation, or particular competence of the specialist

A. Auditor, as a result of the specialist's findings, adds an explanatory paragraph in a modified opinion emphasizing a matter regarding the financial statements When expressing an unmodified opinion, the auditor generally will not refer to the work or findings of a specialist. The auditor may, however, make reference to a specialist in a departure from an unmodified opinion. The auditor may need the permission of the specialist before referencing the specialist in the report.

Under which of the following circumstances would an entity be expected to accrue a loss contingency for the period under audit? A. Before the issuance of the audit report, the entity estimated the amount of a claim that had a probable adverse outcome related to a product sold during the year under audit B. Legal counsel communicated that an unfavorable judgment from current litigation was reasonably possible C. A reasonable estimate was determined for a liability incurred after the balance sheet date D. The entity recorded the amount of an asset impaired as of the balance sheet date

A. Before the issuance of the audit report, the entity estimated the amount of a claim that had a probable adverse outcome related to a product sold during the year under audit Since the underlying event (product sale) relates to the year under audit and it has a probable adverse outcome, the entity should accrue a loss contingency for the period under audit. This is considered a recognized subsequent event. Choice "4" is incorrect. An impairment loss, not a contingency loss, would be recorded for an asset that is impaired as of the balance sheet date. (A contingency is an existing condition, situation, or set of circumstances involving uncertainty as to possible gain or loss that will be ultimately determined when a future event occurs or fails to occur.)

Which of the following procedures would a CPA most likely perform in the planning phase of a f/s audit A. Compare financial info with nonfinancial operating data B. Recalculate the prior year's accruals and deferrals C. Make inquires of the client's lawyer concerning pending litigation D. Perform cutoff tests of cash receipts and disbursements

A. Compare financial info with nonfinancial operating data During the planning phase of an audit, analytical procedures that include comparing financial information to nonfinancial operating data may be performed. This type of test will be used to assist the auditor in understanding the client and its environment, as well as to potentially alert the auditor to problems that could require attention later in the audit.

Which of the following procedures would an auditor generally perform regarding subsequent events? A. Compare the latest available interim financial statements with the statements being audited B. Review the client's cutoff bank statements for several months after the year-end C. Test internal control activities that were previously reported to management as inadequate D. Inspect inventory items that were ordered before the year-end but arrived after the year-end

A. Compare the latest available interim financial statements with the statements being audited

When auditing a client's year-end cash balance, an auditor uses standard bank confirmations and performs tests on the client's year end bank reconciliations. These substantive procedures test which of the following assertions? A. Completeness and valuation and allowance B. Rights and obligations, and occurrence C. Cutoff D. Understandability and classification

A. Completeness and valuation and allowance If an auditor performs tests on year-end bank reconciliations or sends standard bank confirmations to all banks where the client has transacted business during the year, the auditor would be testing the completeness, and valuation and allowance assertions pertaining to the client's ending cash balance. Choice "2" is incorrect. These audit procedures do not test the rights and obligations and occurrence assertion.

Before applying principal substantive tests to the details of accounts at an interim date prior to the balance sheet date, an auditor should: A. Consider whether the amounts of the year-end balances selected for interim testing are reasonable predictable B. Assess control risk at a low level for the assertions embodied in the accounts selected for interim testing C. Determine that the accounts selected for interim testing are not material to the financial statements taken as a whole D. Obtain written representations from management that all financial records and related data will be made available

A. Consider whether the amounts of the year-end balances selected for interim testing are reasonable predictable Before performing substantive tests at an interim date, an auditor should consider whether the amounts of the year-end balances selected for interim testing are reasonably predictable with respect to amount, relative significance and composition.

Restrictions imposed by a client prohibit the observation of physical inventories, which account for 35% of all assets. Alternative audit procedures cannot be applied, although the auditor was able to examine satisfactory evidence for all other items in the financial statements. The auditor should issue a(an): A. Disclaimer of opinion B. Unmodified opinion with an explanation in an emphasis of matter paragraph C. Qualified opinion with a basis for modification paragraph D. Except for qualified opinion

A. Disclaimer of opinion. Restrictions of scope imposed on the audit of such a large (35%) asset would require a disclaimer of opinion.

an auditor who wishes to capture an entity's data as transactions are processed and continuously test the entity's computerized information system most likely would use which of the following techniques? A. Embedded audit module B. Test data generator C. Snapshot application D. Integrated data check

A. Embedded audit module Embedded audit modules are sections of an application program code that collect transaction data for the auditor. Such modules allow the auditor to capture specific data as transactions are being processed.

Analytical Procedures used in the planning phase of an audit should focus on: A. Enhancing the auditor's understanding of the transactions and events that have occurred since the last audit B. Documenting the risk factors relating to the susceptibility of assets to misappropriation C. Discovering uncorrected misstatements that should be communicated to the audit committee D. Identifying the internal control activities that could reduce the assessed level of control risk

A. Enhancing the auditor's understanding of the transactions and events that have occurred since the last audit -Analytical procedures used in the planning phase of an audit should focus on enhancing the auditor's understanding of the transactions and events that have occurred since the last audit. -Identifying the internal control activities that could reduce the assessed level of control risk is part of assessing risk, but it is not related to the performance of analytical procedures.

an auditor most likely would inspect loan agreements under which an entity's inventories are pledged to support management's financial statement assertion of completeness with respect to: I. Presentation & Disclosure II. Transactions & events III. Account Balances A. I Only B. II Only C. III Only D. I, II, and III

A. I Only Inspecting loan agreements under which an entity's inventories are pledged provides evidence regarding completeness with respect to presentation and disclosure, since such information must be disclosed in the financial statements.

An auditor most likely would issue an adverse opinion to: A. Inadequate disclosure of material information B. The inability to determine the extent of or the amounts associated with a pervasive employee fraud scheme C. Management's refusal to provide written representations D. The auditor is unable to obtain the audited financial statements of a consolidated investee.

A. Inadequate disclosure of material information Inadequate disclosure of material information is a departure from GAAP and may result in either a qualified or adverse opinion, depending on materiality. Choice "4" is incorrect. If the auditor is unable to obtain the audited financial statements of a consolidated investee, a qualified opinion or a disclaimer of opinion would be issued, depending on materiality.

Comparative financial statements include the prior year's statements that were audited by a predecessor auditor whose report is not presented. If the predecessor's report was unmodified, the successor should A. Indicate in an other-matter paragraph that the predecessor auditor expressed an unmodified opinion on the prior year's financial statements B. Add an emphasis-of-matter paragraph that expresses only limited assurance concerning the fair presentation of the prior year's financial statements C. Express an opinion only on the current year's financial statements and make no reference to the prior year's financial statements D. Obtain a letter of representation from the predecessor auditor concerning any matters that might affect the successor's opinion

A. Indicate in an other-matter paragraph that the predecessor auditor expressed an unmodified opinion on the prior year's financial statements When a successor auditor does not present the predecessor auditor's report, the successor should indicate in an other-matter paragraph that the predecessor auditor expressed an unmodified opinion on the prior year's financial statements.

Auditors try to identify predictable relationships when using analytical procedures. Relationships involving transactions from which of the following accounts most likely would yield the highest level of evidence? A. Interest expense B. Travel and entertainment expense C. Accounts payable D. Accounts receivable

A. Interest expense Relationships among income statement accounts tend to be more predictable than balance sheet accounts (accounts receivable, accounts payable) because they represent transactions over a period of time rather than at one point in time. In addition, relationships involving transactions subject to management discretion (travel and entertainment) are less predictable.

A client changes from FIFO to LIFO for accounting for inventory and appropriately discloses the change in the footnotes. The Change does not have a material effect on the finanical statements in the current year but the change is expected to have a material effect in later years. In the current-year auditor's report, the auditor should: A. Issue an unmodified opinion B. Issue an unmodified opinion with an emphasis-of-matter paragraph C. Issue a qualified or adverse opinion D. Issue an unmodified opinion with an other-matter paragraph

A. Issue an unmodified opinion If a change in accounting principle, such as a change in accounting for inventory, does not have a material effect on the financial statements in the current year but the change is expected to have a material effect in later years, the auditor is not required to recognize the change in the auditor's report in the current year. Therefore, the auditor should issue an unmodified opinion and does not need to describe the change in an emphasis-of-matter paragraph.

In planning the audit, the auditor obtains a sufficient understanding of the existing internal control. Which one of the following is not among the auditor's primary objectives for obtaining such knowledge? A. Make constructive suggestions to the client for improvement B. Consider the factors that affect the risk of material misstatemnet C. Identify types of material misstatements D. Design effective substantive tests

A. Make constructive suggestions to the client for improvement Making constructive suggestions to the client is not a primary objective for obtaining an understanding of internal control, although it may be a desirable by-product of an audit engagement. Choices "3", "2", and "4" are incorrect, as the following are primary internal control planning objectives of an auditor in a financial statement audit: 3. Identify types of potential material misstatements. 2. Consider factors that affect the risk of material misstatements. 4. Design effective substantive tests.

Which of the following factors most likely would cause a CPA to decide not to accept a new audit engagement? A. Management's disregard of its responsibility to maintain an adequate internal control environment B. The CPA's inability to determine whether related party transactions were consummated on terms equivalent to arm's-length transactions C. Management's refusal to permit the CPA to perform substantive tests before the year-end D. The CPA's lack of understanding of the prospective client's internal auditor's computer-assisted audit techniques

A. Management's disregard of its responsibility to maintain an adequate internal control environment The control environment is the foundation for all other components of internal control. Management's disregard of its responsibility to maintain an adequate internal control environment therefore compromises its ability to provide reasonable assurance regarding reliable financial reporting. The auditor may conclude that the risk that the financial framework used by the client may be unacceptable is great enough that an audit should not be conducted.

Which of the following would a successor auditor ask the predecessor auditor to provide after accepting an audit engagement? A. Matters that may facilitate the evaluation of financial reporting consistency between the current and prior years B. Facts known to the predecessor auditor that might bear on the integrity of management C. The predecessor auditor's understanding of the reasons for the change of auditors D. Disagreements between the predecessor auditor ad management as to significant accounting policies and principles

A. Matters that may facilitate the evaluation of financial reporting consistency between the current and prior years Matters that may facilitate the evaluation of financial reporting consistency between the current and prior years should be discussed after accepting an audit engagement. -Disagreements between the predecessor auditor and management as to significant accounting policies and principles should be discussed prior to accepting an audit engagement. -Information regarding the integrity of management should be discussed prior to accepting an audit engagement.

What effect would the sale of a company's trading securities at their carrying amounts for cash have on each of the following ratios? Current Quick A. No effect No effect B. No effect increase C. increase No effect D. Increase increase

A. No effect No effect Trading securities are both current assets and quick assets. If they are sold for their carrying value, then both total current assets and total quick assets remain constant since one type of current asset and quick asset is traded for another. Thus, both the current ratio and the quick ratio would be unaffected by the sale of trading securities.

When there has been a change in accounting principles, but the effect of the change on the comparability of the financial statements is not material, the auditor should A. Not refer to consistency in the auditor's report B. Refer to the change in the opinion paragraph C. Explicitly concur that the change is preferred D. Refer to the change in an emphasis-of-matter paragraph

A. Not refer to consistency in the auditor's report If an accounting change has no material effect on the comparability of the financial statements, the auditor does not need to recognize the change in the current year's audit report.

A test of a payroll system involved comparing an individual's number of overtime hours a week with an average of weekly overtime during a similar period in a prior year and evaluating the results. This is an example of what type of test? A. Reasonableness test B. Category Test C. Range test D. Detail test

A. Reasonableness test In a reasonableness test, data in two or more fields are checked for consistency. Comparing overtime hours in the current period to a prior period is one type of reasonableness test. Choice "4" is incorrect. A test of details is one in which specific details are evaluated, whereas in a reasonableness test, two different fields are compared. A test of details with respect to overtime hours might involve looking at time sheets for that week, for example.

An auditor should be aware of subsequent events that provide evidence concerning conditions that did not exist at year end but arose after year end. These events may be imporant to the auditor because they may: A. Require disclosure to keep the financial statements from being misleading B. Require adjustments to the financial statements as of the year end C. Have been recorded based on year-end tests for asset obsolescence D. Have been recorded based on preliminary accounting estimates

A. Require disclosure to keep the financial statements from being misleading Conditions that did not exist at year end but arose after year end are Type 2 (nonrecognized) subsequent events that must be disclosed to keep the financial statements from being misleading. Choice "2" is incorrect. An adjustment would not be required as the conditions did not exist at year end. Type 1 (recognized) subsequent events require adjustment to the financial statement.

Which of the following procedures would least likely result in the discovery of possible noncompliance with laws and regulations? A. Reviewing an internal control questionnaire B. Reading the minutes of the board of directors' meetings C. Making inquires of management or legal counsel D. Performing tests of details of transactions

A. Reviewing an internal control questionnaire Reviewing an internal control questionnaire provides information about control policies and procedures, but does not provide information about actual transactions or events that have occurred. Therefore, it is not likely to uncover any acts of noncompliance with laws and regulations.

Which of the following procedures would an auditor least likely perform in planning a financial statement audit? A. Selecting a sample of vendors' invoices for comparison to receiving reports B. Reading the current year's interim financial statements C. Discussing matters that may affect the audit with firm personnel responsible for non-audit services to the entity D. Coordinating the assistance of entity personnel in data preparation

A. Selecting a sample of vendors' invoices for comparison to receiving reports Selecting a sample of vendors' invoices for comparison to receiving reports is performed during fieldwork. This is not part of the planning phase.

which of the following courses of action would an auditor most likely follow in planning a sample of cash disbursements if the auditor is aware of several unusually large cash disbursements? A. Stratify the cash disbursements population so that the unusually large disbursements are selected B. Set the tolerable rate of deviation at the lower level than originally planned C. Continue to draw new samples until all the unusually large disbursements appear in the sample D. Increase the sample size to reduce the effect of the unusually large disbursements

A. Stratify the cash disbursements population so that the unusually large disbursements are selected stratification involves the grouping of transactions sharing some characteristic (such as recorded amounts). The goal of stratification is to ensure selection of items for which potential misstatements may individually equal or exceed tolerable misstatement. Thus, the auditor should stratify the sample such that the unusually large transactions are selected

During an audit of a nonissuer's financial statements, an auditor should perform tests of controls to obtain sufficient appropriate audit evidence about the operating effectiveness of relevant controls if: A. Substantive procedures alone cannot provide sufficient appropriate audit evidence B. The auditor does not presume that the client management has committed fraud C. The auditor does not intend to rely on the operating effectiveness of controls D. More financial documentation is available through tests of controls

A. Substantive procedures alone cannot provide sufficient appropriate audit evidence The auditor should perform tests of controls when substantive procedures alone cannot provide sufficient appropriate audit evidence. Choice "3" is incorrect. If the auditor does not intend to rely on the operating effectiveness of controls, then the auditor does not need to perform tests of controls.

If an auditor is obtaining an understanding of an issuer's information and communication component of internal control, which of the following factors should the auditor assess? A. The classes of transactions in the issuer's operations that are significant to the issuer's financial statements B. The philosophy and operating style of management to promote effective internal control over financial reporting C. The oversight responsibility over financial reporting and internal control by the board or audit committee D. The integrity and ethical values of top management

A. The classes of transactions in the issuer's operations that are significant to the issuer's financial statements The classes of transactions in the issuer's operations that are significant to the issuer's financial statements are typically assessed when the auditor is obtaining an understanding of the information and communication component of internal control.

As a result of sampling procedures applied as tests of controls, an auditor incorrectly assesses control risk lower than appropriate. The most likely explanation for this situation is that A. The deviation rate in the auditor's sample is less than the tolerable rate, but the deviation rate in the population exceeds the tolerable rate B. The deviation rates of both the auditor's sample and the population is less than the tolerable rate C. The deviation rate in the auditor's sample exceeds the tolerable rate, but the deviation rate in the population is less than the tolerable rate D. The deviation rates of both the auditor's sample and the population exceed the tolerable rate

A. The deviation rate in the auditor's sample is less than the tolerable rate, but the deviation rate in the population exceeds the tolerable rate The situation given is an example of sampling risk, or the probability that the sample chosen is not representative of the population as a whole. This is illustrated by the fact that the sample deviation rate is less than the tolerable rate, but the true deviation rate exceeds the tolerable rate.

which of the following characteristics most likely would heighten an auditor's concern about the risk of material misstatements in an entity's financial statements? A. The entity's industry is experiencing declining customer demand B. Equipment is often sold at a loss before being fully depreciated C. Bank reconciliations usually include in-transit deposits D. Employees who handle cash receipts are not bonded

A. The entity's industry is experiencing declining customer demand In assessing the risk related to material misstatements in an entity's financial statements, the auditor would consider situations that threaten financial stability or profitability, since such situations might provide an incentive to fraudulently misstate the financial statements. Included as one of these characteristics is declining customer demand.

A client decides not to make an auditor's proposed adjustments that collectively are not material, and wants the auditor to issue the report based on the unadjusted numbers. Which of the following statements is correct regarding the financial statement presentation? A. The f/s are free from material misstatement, and no disclosure is required in the notes to the f/s B. The f/s do not conform with GAAP C. The f/s contain unadjusted misstatements that should result in a qualified opinion D. The f/s are free from material misstatement, but disclosure of the proposed adjustments is required in the notes to the f/s

A. The f/s are free from material misstatement, and no disclosure is required in the notes to the f/s An unmodified opinion states that the financial statements are presented fairly, in all material respects. Since the collective effect of the proposed adjustments is immaterial, an unmodified opinion should be expressed. In addition, footnote disclosure of proposed immaterial adjustments is not required.

Which of the following auditor concerns most likely could be so serious that the auditor concludes that a financial statement audit cannot be performed? A. There is a substantial risk of intentional misapplication of accounting principles B. Management is dominated by one person who is also the majority stockholder C. Internal control activities requiring segregation of duties are rarely monitored by management D. Management fails to modify prescribed internal controls for changes in information technology

A. There is a substantial risk of intentional misapplication of accounting principles Intentional misapplication of accounting principles would indicate that management lacks integrity and as a result, the auditor might conclude that a financial statement audit cannot be performed. -If management is dominated by one person who is also the majority stockholder, the risk of fraudulent financial reporting is increased, but this would not preclude the auditor from performing a financial statement audit.

As part of developing an audit strategy for an existing client, the auditor may determine a materiality level for all the following with the exception of: A. Transaction cycles with misstatements in the prior year's audit B. Certain classes of transactions C. Performance materality D. Financial statements as a whole

A. Transaction cycles with misstatements in the prior year's audit While developing an audit strategy, the auditor determines the level of materiality for the client's financial statements as a whole, performance materiality, and if necessary, for certain account balances, transactions, and disclosures. A separate materiality level would not be established for specific transaction cycles that contained misstatements in the prior year's audit.

When performing analytical procedures in the planning stage, the auditor most likely would develop expectations by reviewing which of the following sources of information? A. Unaudited information from internal quarterly reports B. Comments in the prior-year management letter C. Various account assertions in the planning memorandum D. The control risk assessment relating to specific financial assertions

A. Unaudited information from internal quarterly reports Analytical procedures involve comparison of recorded amounts to independent expectations developed by the auditor. During the planning stage, analytical procedures generally use financial data, such as unaudited information from internal quarterly reports. Choice "3" is incorrect. Analytical procedures involve comparison of recorded amounts to independent expectations developed by the auditor. Various account assertions in a planning memorandum would not necessarily be helpful in developing expectations.

An auditor's purpose in reviewing the renewal of a note payable shortly after the balance sheet date most likely is to obtain evidence concerning management's assertions about: A. Understandability & Classification B. Occurrence C. Completeness D. Valuation & accuracy

A. Understandability & Classification An auditor's purpose in reviewing the renewal of a note payable shortly after the balance sheet date most likely is to obtain evidence concerning management's assertions about understandability and classification (i.e., classification of the note as current or noncurrent). Choice "3" is incorrect. Completeness relates to whether all transactions occurring in the period have been recorded. Since the renewal took place subsequent to year-end, it is not relevant to the completeness assertion for the year under audit.

Park, CPA, was engaged to audit the financial statements of Tech Co., a new client, for the year ended December 31, Year 1. Park obtained sufficient appropriate audit evidence for all of Tech's financial statement items except Tech's opening inventory. Due to inadequate financial records, Park could not verify Tech's January 1, Year 1, inventory balances. Park's opinion on Tech's Year 1 financial statements most likely will be Balance sheet Income Statement A. Unmodified Disclaimer B. Unmodified Adverse C. Disclaimer Adverse D. Disclaimer Disclaimer

A. Unmodified Disclaimer When the auditor is unable to satisfy himself or herself regarding the amount of beginning inventory, he or she must disclaim an opinion on the income statement because of the inability to verify the cost of goods sold during the year. The auditor may, however, still be able to issue an unmodified opinion on the balance sheet, since inventory can be verified as of the balance sheet date.

As a part of audit procedures performed on a client's transactions, the auditor reconciles supporting schedules to the corresponding general ledger entries. The auditor would most likely be testing which of the following assertions? A. Valuation, allocation, accuracy B. Understandability and classification C. Existence and occurrence D. Cutoff

A. Valuation, allocation, accuracy If the auditor performs a reconciliation of supporting schedules to the client's general ledger entries, the auditor would most likely be testing the valuation, allocation, and accuracy assertion for a particular transaction cycle.

Which of the following is not an inquiry the auditor should make to identify the risks of material misstatement due to fraud? A. Whether operating personnel have communicated to management regarding internal control and how it functions to prevent, deter, or detect material misstatement due to fraud B. Whether there are any particular business segments for which a risk of fraud may be more likely to exist C. How management communicates to employees its views on acceptable business practices D. Whether management is aware of any allegations of fraud

A. Whether operating personnel have communicated to management regarding internal control and how it functions to prevent, deter, or detect material misstatement due to fraud The auditor should inquire whether management (not operating personnel) has communicated to those charged with governance (not management) regarding internal control and how it functions to prevent, deter, or detect material misstatement due to fraud.

Which of the following documents the procedures that are applied and the conclusions reached in an audit engagement? A. Working papers B. Management representation letter C. Audit Guide D. Auditor's report

A. Working papers Working papers should include the procedures that are applied and the conclusions reached in an audit engagement.

The audit plan usually cannot be finalized until the: A. consideration of the entity's internal control has been completed B. Search for unrecorded liabilities has been performed and documented C. Significant deficiencies in internal control have been communicated to those charged with governance D. Representation letter has been signed by the client

A. consideration of the entity's internal control has been completed The auditor should obtain a sufficient understanding of the entity and its environment, including its internal control, to plan the audit of the entity's financial statements.

In Which of the following situations would an auditor ordinarily choose between expressing an "except for" qualified opinion or an adverse opinion? A. The auditor did not observe the entity's physical inventory and is unable to become satisfied as to its balance by other auditing procedures B .The f/s fail to disclose information that is required by GAAP C. Events disclosed in the financial statements cause the auditor to have substantial doubt about the entity's ability to constitute as a going concern D. The auditor is asked to report only on the entity't balance sheet and not on the other basic financial statements

B .The f/s fail to disclose information that is required by GAAP Failure to disclose information that is required by GAAP is a departure from GAAP. Departures from GAAP result in a qualified or an adverse opinion.

As part of the current year's audit, the auditor sends out client customer confirmations and receives back a number of confirmation responses with exceptions. After investigating further, the auditor would be most concerned with which of the following reasons for the confirmation exceptions? A. There was a measurement difference between the client and responding customer B. A deficiency in accounts receivable internal control resulted in client reporting errors C. A timing difference cause the exception D. There was a clerical error in the confirmation made by the auditor

B. A deficiency in accounts receivable internal control resulted in client reporting errors An auditor would be most concerned if the confirmation exceptions were due to a client internal control weakness over reporting, a material misstatement was discovered, or fraud occurred Choice "1" is incorrect. A measurement difference is of less concern to the auditor as it does not represent a material misstatement.

Which of the following situations represents a limitation, rather than a failure, of internal control? A. A movie theater cashier sells reduced-price tickets to full-paying customers and pockets the difference B. A purchasing employee and an outside vendor participate in a kickback scheme C. A bank teller embezzles several hundred dollars from the cash drawer D. A jewelry store employee steals a small necklace from a display cabinet

B. A purchasing employee and an outside vendor participate in a kickback scheme -Even a well-designed internal control system has its limitations. One example of a limitation of internal control includes deliberate circumvention of controls by collusion of two or more people, such as when a purchasing employee and an outside vendor participate in a kickback scheme. Other limitations of internal controls include human error and management override of control.

Which of the following is not true regarding an engagement to provide a written report on the application of the requirements of an applicable financial reporting framework? A. The reporting accountant's written report on the application of the requirements of the application of the applicable financial reporting framework should include a paragraph restricting the use of the report B. A reporting accountant is prohibited from providing a report on the application of the requirements of an applicable financial reporting framework to a proposed future transaction involving the facts and circumstances of a specific entity C. The reporting accountant's written report on the application of the requirements of an applicable financial reporting framework should include an identification of the specific entity involved D> A reporting accountant is prohibited from providing a report on the application of the requirements of an applicable financial reporting framework to a transaction not involving the facts and circumstances of a specific entity.

B. A reporting accountant is prohibited from providing a report on the application of the requirements of an applicable financial reporting framework to a proposed future transaction involving the facts and circumstances of a specific entity -A reporting accountant may report on the application of the requirements of an applicable financial reporting framework to a proposed future transaction as long as the transaction involves the facts and circumstances of a specific entity. -The reporting accountant's written report on the application of an applicable financial reporting framework should include an identification of the specific entity involved; a brief description of the nature of the engagement; a statement that the engagement was performed in accordance with AICPA standards; a description of the specific transaction(s); a statement of the relevant facts, circumstances, assumptions and source of the information; a statement describing the appropriate application of the requirements of the applicable financial reporting framework to the specific transaction or type of report; a statement that the preparers of the financial statements are responsible for proper accounting treatment; a statement that any difference in facts, circumstances or assumptions presented may change the report; a separate paragraph restricting its use to specified parties; and a statement indicating that the reporting accountant is not independent (if appropriate)

Which of the following is true about modifications to the independent auditor's unmodified opinion report for a nonissuer? A. Modifications to the independent auditor's report result in qualified, adverse or negative assurance opinions, or a disclaimer of opinion B. An auditor would modify the same paragraphs when rendering either a qualified opinion due to a departure from GAAP or a qualified opinion due to a scope limitation C. A disclaimer of opinion and an adverse opinion both include modification to the introductory paragraph D. An auditor would modify different paragraphs when rendering either a qualified opinion due to a departure from GAAP or an adverse opinion due to a departure from GAAP

B. An auditor would modify the same paragraphs when rendering either a qualified opinion due to a departure from GAAP or a qualified opinion due to a scope limitation A qualified opinion due to a scope limitation and a qualified opinion due to a GAAP departure require a paragraph preceding the opinion paragraph titled "Basis for Qualified Opinion".

Which of the following statements regarding risk assessment is correct? A. The risk of material misstatement is a function of detection risk and control risk B. An auditor's assessment of control risk at a low level can help to offset a higher level of inherent risk C. An auditor is required to assess the inherent risk, control risk, and detection risk within the client before beginning the audit D. A high assessment of detection risk will require a reduction in inherent risk

B. An auditor's assessment of control risk at a low level can help to offset a higher level of inherent risk Control risk and inherent risk are multiplied together to determine the overall risk of material misstatement. Thus, an assessment of control risk at a low level will help to reduce the overall risk of material misstatement.

Audit documentation should be prepared in enough detail so that: A. A reader of the financial statements who has no previous connection with the audit can understand the procedures performed and the evidence obtained B. An experienced auditor who has no previous connection with the audit can understand the procedures performed and the evidence obtained C. An experienced auditor who has worked with the client in the pas can understand the procedures performed and the evidence obtained D. A reader of the financial statements who has a background in financial analysis can understand the procedures performed and the evidenced obtained

B. An experienced auditor who has no previous connection with the audit can understand the procedures performed and the evidence obtained

If a component auditor does not meet the independent requirements that are relevant to a group audit of a nonissuer's financial statements, then the group engagement team should first: A. Communicate the lack of independence to the appropriate regulatory authority B. Attempt to obtain sufficient appropriate audit evidence relating to the financial information of the component without making reference to or using the work of the component auditor C. Disclose the lack of Independence to the nonissuer's management and consider revising the audit report D. Withdraw from the engagement when permissible under law or regulation

B. Attempt to obtain sufficient appropriate audit evidence relating to the financial information of the component without making reference to or using the work of the component auditor If a component auditor does not meet the independence requirements that are relevant to a group audit of a nonissuer's financial statements, then the group engagement team should first attempt to obtain sufficient appropriate audit evidence relating to the financial information of the component without making reference to or using the work of the component auditor.

The auditor's report should include reference to the US as the country of origin of: I The accounting principles used to prepare the financial statements II. The auditing standards the auditor followed in performing the audit A. Neither I or II B. Both I and II C. II only D. I only

B. Both I and II The auditor's report should include reference to the United States as the country of origin of both the accounting principles used to prepare the financial statements and the auditing standards the auditor followed in performing the audit.

Detection risk differs from both control risk and inherent risk in that detection risk: A. Should be assessed in non quantitative term B. Can be changed at the auditor's discretion C. Arises from risk factors relating to fraud D. Exists independently of the f/s audit

B. Can be changed at the auditor's discretion Detection risk can be changed at the auditor's discretion, whereas control risk and inherent risk exist independently of the financial statement audit, and cannot be changed by the auditor.

in which of the following situations would an auditor ordinarily choose between expressing a qualified opinion or an adverse opinion? A. The auditor did not observe the entity's physical inventory & is unable to become satisfied about its balance by other auditing procedures B. Conditions that cause the auditor to have substantial doubt about the entity's ability to continue as a going concern are inadequately disclosed C. There has been a change in accounting principles that has a material effect on the comparability of the entity's financial statements D. The auditor is unable to apply necessary procedures concerning an investor's share of an investee's earnings recognized on the equity method

B. Conditions that cause the auditor to have substantial doubt about the entity's ability to continue as a going concern are inadequately disclosed Inadequate disclosure of the substantial doubt about an entity's ability to continue as a going concern is a departure from GAAP, resulting in either a qualified or adverse opinion. Choice "3" is incorrect. A change in accounting principle results in the use of an emphasis-of-matter paragraph following the opinion paragraph in an unmodified report, as long as the change was accounted for properly.

Which audit procedure is most likely related to the classification & understandability of the financial statements with respect to inventory? A. Examining consignment agreements B. Confirming inventories pledged under loan agreements C. Obtaining quotations for the current market value of inventory D. Analyzing inventory turnover

B. Confirming inventories pledged under loan agreements The pledge or assignment of any inventories should be appropriately disclosed in the financial statements. Choice "1" is incorrect. Consignment agreements should be examined to ensure that the entity has ownership rights to the inventory (and does not include inventory owned by others), but this relates to the rights and obligations assertion, not to classification and understandability assertions.

Which of the following strategies would a CPA most likely consider in auditing an entity that processes most of its financial data only in electronic form, such as a paperless system? A. Increased reliance on internal control activities that emphasize the segregation of duties B. Continuous monitoring and analysis of transaction processing with an embedded audit module C. Verification of encrypted digital certificates used to monitor the authorization of transactions D. Extensive testing of firewall boundaries that restrict the recording of outside network traffic

B. Continuous monitoring and analysis of transaction processing with an embedded audit module In a paperless system, the CPA would most likely need to continuously monitor and analyze transaction processing to ensure that controls were operating effectively throughout the period under audit. An embedded audit module is one way to achieve this goal.

In its tests of controls over the Tarbet Township Housing Assistance Program, Smith, CPA, has found that the clerk assigned to monitor and limit participation in the house program to a target population of individuals meeting income criteria is routinely overruled by his supervisor in order to meet volume based level of effort requirements. Ineligible participants whose income exceeds program limits are routinely admitted to the program. Smith would characterize this as a(n): A. Deficiency int eh design of internal control B. Deficiency in the operation of internal control C. Audit risk of noncompliance D. Inherent risk of noncompliance

B. Deficiency in the operation of internal control Deficiency in operation exists when a properly designed control is either not executed as designed or the person performing the control does not have either the authority or the skill to perform the control. In this case, the clerk did not have the necessary authority to follow through on the control.

Which of the following procedures is an auditor least likely to perform if material disclosures required by GAAP are omitted? A. Discuss the omission of such information with those charged with governance B. Disclose the omitted information in the notes to the financial statements C. Discuss the omission of such information with management D. Disclose the omitted information in the basis-for-modification paragraph

B. Disclose the omitted information in the notes to the financial statements Management is responsible for the financial statements. The auditor may include information within the auditor's report, but may not include information within the financial statements and the related notes. Choice "4" is incorrect. When material disclosures required by GAAP are omitted, the auditor should disclose the omitted information in the basis-for-modification paragraph.

Analytical Procedures used in the planning phase of an audit should focus on: A. Identifying material weaknesses in internal control B. Enhancing the auditor's understanding of the client's business C. Testing individual account balances that depend on accounting estimates D. Evaluating the adequacy of evidence gathered concerning unusual balances

B. Enhancing the auditor's understanding of the client's business Choice "2" is correct. The purpose of applying analytical procedures in planning the audit is to assist in planning the nature, timing, and extent of auditing procedures that will be used to obtain audit evidence for specific account balances or classes of transactions. To accomplish this, the analytical procedures used in planning the audit should focus on (a) enhancing the auditor's understanding of the client's business and the transactions and events that have occurred since the last audit date, and (b) identifying areas that may represent specific risks relevant to the audit. Choice "4" is incorrect. Analytical procedures to assess the adequacy of evidence would be used in the final review stage.

Which of the following is a management control method that most likely could improve management's ability to supervise company activities effectively? A. Supporting employees with the resources necessary to discharge their responsibilities B. Establishing budgets and forecasts to identify variances from expectations C. Monitoring compliance with internal control requirements imposed by regulatory bodies D. Limiting direct access to assets by physical segregation and protective devices

B. Establishing budgets and forecasts to identify variances from expectations The use of budgets and forecasts to identify variances improves managers' ability to supervise company activities. Variances from budgets serve as signals to managers that a potential problem exists. Compliance with internal controls mandated by a regulatory body may be necessary, but would not necessarily enhance the ability of a manager to supervise company activities.

An auditor's report under US auditing standards that refers to a scope limitation that is material but not pervasive contains the words," In our opinion, except for the above mentioned limitation on the scope of our audit...." This is considered a(n): A. Adverse opinion B. Example of inappropriate wording C. Disclaimer of opinion D. Qualified opinion

B. Example of inappropriate wording "In our opinion, except for the above-mentioned limitation on the scope of our audit . . ." is an example of inappropriate wording. When the auditor expresses a qualified opinion due to a scope limitation, the auditor should state in the opinion paragraph that the qualification pertains to the possible effects of the matter on the financial statements and not to the scope limitation itself. The wording used should be, "In our opinion, except for the possible effects of the matter described in the basis-for-qualified-opinion paragraph . . . ."

In testing plant & equipment balances, an auditor examines new additions listed on an analysis of plant & equipment. This procedure most likely obtains evidence concerning management's assertion of: A. Completeness B. Existence C. Understandability & classification D. Valuation & allocation

B. Existence By examining the assets listed as new additions on an analysis of plant and equipment, an auditor obtains evidence concerning management's assertion of existence. Choice "1" is incorrect. Examining new additions that are already listed on the analysis does not provide evidence of completeness. The assertion of completeness relating to fixed assets might be tested by reviewing large repair and maintenance charges to determine if the cost should more properly have been capitalized (i.e., to evaluate whether fixed asset additions are complete).

The auditor is planning to use accounts payable confirmations when auditing the client's accounts payable. The use of accounts payable confirmations will test which of the following assertions? A. Rights and obligations B. Existence and completeness C. Cutoff D. Existence (only)

B. Existence and completeness While accounts payable confirmations are generally used to test the existence assertion, it is also considered a test of the completeness assertion

Zag Co. issues financial statements that present financial position and results of operations but Zag omits the related statement of cash flows. Zag would like to engage Brown, CPA, to audit its financial statements without the statement of cash flows although Brown's access to all of the information underlying the basic financial statements will not be limited. Under these circumstances, Brown most likely would: A. Refuse to accept the engagement as proposed becausse of the client-imposed scope limitation B. Explain to Zag that the omission requires a qualification of the auditor's opinion C. Add an emphasis-of-matter paragraph to the auditor's report that justifies the reason for the omission. D. Prepare the statement of cash flows as an accommodation to Zag and express an unmodified opinion.

B. Explain to Zag that the omission requires a qualification of the auditor's opinion The auditor would explain to the client that in order for the entity's financial statements to be in conformity with GAAP, there must be adequate disclosures of all material matters including all financial statements and the supporting footnotes. As a result, the auditor would tell Zag that without adequate disclosure of the entity's cash flows, the audit report would have to be issued with a qualified or adverse audit opinion.

which of the following levels would most likely address the risk of material misstatement by the auditor's consideration of an entity's control environment? A. Disclosures B. Financial statements C. Classes of transactions D. Specific account balances

B. Financial statements The control environment is pervasive and reflects the overall tone of the organization. Therefore, the auditor is most likely to focus on the highest level of risk of material misstatement, which is risk assessed at the financial statement level.

By obtaining an understanding of the client and its internal control environment, the auditor is directly assessing the client's: A. Inherent risk B. Inherent and control risk C. Control risk D. Detection risk

B. Inherent and control risk The risk of material misstatement is driven by inherent risk and control risk. So if the auditor obtains an understanding of the client and its environment including its internal controls, the auditor is effectively assessing both the client's inherent risk and control risk.

Equipment acquisitions that are misclassified as maintenance expense most likely would be detected by an internal control activity that provides for: A. Independent verification of invoices for disbursements recorded as equipment acquisitions B. Investigation of variances within a formal budgeting system C. Authorization by the board of directors of significant equipment acquisitions D. Segregation of duties of employees in the accounts payable department

B. Investigation of variances within a formal budgeting system Equipment acquisitions that are misclassified as maintenance expense most likely would be detected by internal control procedures that provide for investigation of variances within a formal budgeting system. Choice "1" is incorrect. Verifying invoices for disbursements already recorded as equipment acquisitions would not include examining invoices for disbursements recorded as maintenance expense.

Which of the following is not a primary characteristic associated with audit risk? A. Audit risk arises if the auditor obtains only reasonable assurance that the client's financial statements are not materially misstated B. It is a risk that the auditor may fail to detect a client's deceptive practices which may result in a misstatement of the financial statements C. Before an opinion is provided for an audit, the audit risk should be reduced to an appropriately low level D. It is the risk associated with the auditor failing to appropriately modify an audit opinion when the client's financial statements contain material misstatements

B. It is a risk that the auditor may fail to detect a client's deceptive practices which may result in a misstatement of the financial statements . This does not represent a characteristic of audit risk. Failing to detect deceptive practices (i.e., management collusion) that may result in the client's financial statement being materially misstated is associated with fraud risk. Choice "1" is incorrect. If the auditor obtains reasonable (and not absolute) assurance that the financial statements are not materially misstated, audit risk still exists.

While performing audit fieldwork on a client, the auditor reviews the client's depreciation policies and schedules on a select group of fixed assets. After careful review of the estimated lives on these fixed assets, the auditor determines that the basis used to determine the length of the estimated useful lives for certain fixed assets is unreasonable and not supported by any internal or industry factors. Based on the above, the auditor would most likely consider this a: A. Projected misstatement B. Judgmental misstatement C. Fraudulent misstatement D. Factual misstatement

B. Judgmental misstatement The scenario above would be considered a judgmental misstatement. The accounting estimates used by management for certain fixed assets are considered unreasonable by the auditor and not supported by any internal or external (industry) rationale. The auditor would consider this a judgmental misstatement as it involves an unreasonable estimate by the client.

Taylor is an employee of Green Solutions. Leah, CPA is asked to express an opinion on Green Solutions' stockholders' equity. According to US auditing standards, Leah may accept this engagement only if: A. Leah also audits Green Solutions' income statement and balance sheet. B. Leah also audits Green Solutions' balance sheet C. The report is used to obtain a loan from a bank D. Leah also is engaged to audit the complete set of financial statements of Green Solutions

B. Leah also audits Green Solutions' balance sheet When a specific element is stockholders' equity, Leah should perform procedures necessary to express an opinion about the balance sheet because of the interrelationship between stockholders' equity and the balance sheet accounts.

An auditor most likely would issue a disclaimer of opinion because of: A. A Material departure from GAAP B. Management's refusal to furnish written representations C. The omission of the statement of cash flows D. Inadequate disclosure of material information

B. Management's refusal to furnish written representations Management's refusal to furnish written representations is a significant client imposed restriction on the scope of an audit, ordinarily warranting a disclaimer of opinion. Choice "3" is incorrect. A qualified report would be appropriate when a "statement of cash flows" is omitted and the scope of the audit is not restricted.

Which of the following phrases should be included in the opinion paragraph when an auditor expresses a qualified opinion? When read in conjunction with Note X With the foregoing explanation A. Yes No B. No Yes C. Yes Yes D. No No

D. No No A qualified opinion phrase is, "in our opinion, except for [explanation of problem] as discussed in the preceding (non-issuer)/following (issuer) paragraph . . ."

While auditing the financial statements of a nonissuer, a CPA was requested to change the engagement to a review in accordance with Statements on Standards for Accounting and Review Services (SSARS) because of a scope limitation. If the CPA believes the client's request is reasonable, the CPA's review report should: I. Refer to the scope limitation that caused the change II. Describe the auditing procedures that have already been applied A. Both I and II B. Neither I nor II C. II only D. I only

B. Neither I nor II If the CPA believes the client's request is reasonable, he/she must comply with the standards for a review and issue an appropriate report. The report should not refer to the original engagement, to any auditing procedures performed, or to the scope limitation

which of the following procedures would an auditor most likely perform before the balance sheet date? A. Confirm with client's lawyer that all litigation probable of assertion has been disclosed to the auditor B. Obtain an understanding of the client's internal control activities C. Determine whether there are any liens or encumbrances on assets that have been pledged as collateral D. Consider the client's plans and ability to meet imminent purchase commitments and cash flow obligations

B. Obtain an understanding of the client's internal control activities An auditor most likely would obtain an understanding of the client's internal control activities before the balance sheet date. This is typically performed in the planning stage of the audit. Choice "1" is incorrect. The auditor confirms with the client's lawyer that all litigation probable of assertion has been disclosed to the auditor after the balance sheet date. The auditor generally wants the lawyer's response dated close to the date of the auditor's report.

In an audit of financial statements for which an auditor's assessment of risk is judgmental and may not be sufficiently precise to identify all risks of material misstatement, the auditor should take which of the following actions? A. Discuss strategies to eliminate such risks with top management or those with equivalent authority and responsibility B. Perform substantive procedures for all relevant assertions related to each material class of transactions C. Determine the effectiveness of general controls over classes of transactions characterized by a high transaction volume D. Consider whether risk assessment procedures are appropriate given preliminary levels of materiality and tolerable misstatement

B. Perform substantive procedures for all relevant assertions related to each material class of transactions In an audit of financial statements, substantive procedures will always be necessary for all relevant assertions related to material transaction classes. Choice "4" is incorrect. Considering the appropriateness of risk assessment procedures does not adequately respond to the fact that the auditor's assessment of risk is always subjective rather than objective. The appropriate response is to perform substantive procedures for all relevant assertions related to each material class of transactions

An auditor of a nonissuer should disclose the reasons for an inability to obtain sufficient appropriate audit evidence in a basis-for-modification paragraph A. Preceding the auditor's responsibility paragraph B. Preceding the opinion paragraph C. Following the opinion paragraph D. Within the notes to the financial statements

B. Preceding the opinion paragraph An auditor should disclose the reasons for an inability to obtain sufficient appropriate audit evidence in a basis-for-modification paragraph preceding the opinion paragraph.

An auditor of a nonissuer should disclose the substantive reasons for expressing an adverse opinion in a basis for modification paragraph: A. Preceding the introductory paragraph B. Preceding the opinion paragraph C. Following the opinion paragraph D. Within the notes to the financial statements

B. Preceding the opinion paragraph The auditor should disclose the substantive reasons for expressing an adverse opinion in a separate basis for adverse opinion paragraph preceding the opinion paragraph.

An auditor should obtain sufficient knowledge of an entity's information system relevant to financial reporting to understand the: A. Safeguards used to limit access to computer facilities B. Process used to prepare significant accounting estimates C. Policies used to detect the concealment of irregularities D. Procedures used to assure proper authorization of transactions

B. Process used to prepare significant accounting estimates An auditor is responsible for evaluating the reasonableness of significant accounting estimates made by management. An entity's information system may affect the quality of such estimates and therefore should be considered by the auditor. -Control activities such as those designed to limit access, ensure proper authorization, and discover fraud are not directly related to the information system relevant to financial reporting.

If a publicly held company issues financial statements that purport to present its financial position and results of operations but omits the statement of cash flows, the auditor ordinarily will express a(an): A. Review report B. Qualified opinion C. Disclaimer of Opinion D. Unmodified opinion with an emphasis-of-matter paragraph

B. Qualified opinion If a company issues financial statements that purport to present financial position and results of operations but omits the related statement of cash flows, the auditor will normally conclude that the omission requires qualification of the opinion.

Risks relevant to financial reporting can arise due to which of the following circumstances? A. Performance reviews of employees B. Rapid growth in the entity's operations C. Corrective actions implemented by management D. Board of directors' commitment to competence

B. Rapid growth in the entity's operations If management implements corrective actions, this likely would result in a decrease in risks relevant to financial reporting.

An auditor's report for financial statements prepared using the special purpose framework of the cash basis accounting contains the following title and sentences: Basis of Accounting We draw attention to Note X of the financial statements, which describes the basis of accounting. The financial statements are prepared on the cash basis of accounting which is a basis of accounting other than accounting principles generally accepted in the US. Our opinion is not modified with respect to this matter A. Represent an other-matter paragraph B. Represent an emphasis-of-matter paragraph C. Should appear prior to the opinion paragraph and explain the reason for a modified opinion D. Are an improper form of reporting

B. Represent an emphasis-of-matter paragraph The title and sentences represent an emphasis-of-matter paragraph. An emphasis-of-matter paragraph is required when the financial statements are prepared in accordance with an applicable special purpose framework, such as the cash basis of accounting. Note that an emphasis of matter paragraph may use the heading of "Emphasis of Matter" or any other appropriate heading.

An auditor reads the letter of transmittal accompanying a county's comprehensive annual financial report and identifies a material inconsistency with the financial statements. The auditor determines that the financial statements do not require revision. Which of the following actions should the auditor take? A. Include an other-matter paragraph in the auditor's report B. Request that the client revise the letter of transmittal C. Consider withdrawing from the engagement D. Request a client representation letter acknowledging the inconsistency

B. Request that the client revise the letter of transmittal When information accompanies audited financial statements in a client-prepared document, the auditor is required to read the information. If such information is materially inconsistent with the financial statements and the financial statements do not require revision, the auditor should request that the information (in this case the letter of transmittal) be revised.

Before a predecessor auditor reissues the prior year's audit report on the financial statements of a former client for inclusion with the successor auditor's report on comparative financial statements, the predecessor does all the following except: A. Obtain a successor auditor representation letter B. Review the audit documentation of the successor auditor C. Obtain the current comparative financial statements D. Compare the current period comparative financial statements with those of the prior year

B. Review the audit documentation of the successor auditor

In obtaining an understanding of a manufacturing entity's internal control concerning inventory balances, an auditor most likely would: A. Perform test counts of inventory during the entity's physical count B. Review the entity's description of inventory controls C. Analyze monthly production reports to identify variances and unusual transactions D. Analyze inventory turnover statistics to identify slow-moving and obsolete items

B. Review the entity's description of inventory controls In obtaining an understanding of a manufacturing entity's internal control concerning inventory balances, an auditor would most likely review the entity's descriptions of inventory controls.

A CPA is engaged to examine management's assertion that the entity's schedule of investment returns is presented in accordance with specific criteria. In performing this engagement, the CPA should comply with the provisions of: A. Statements on Auditing Standards (SAS) B. Statements on Standards for Attestation Engagements (SSAE) C. Statements on Standards for Consulting Services (SSCS) D. Statements on Standards for Accounting and Review Services (SSARS)

B. Statements on Standards for Attestation Engagements (SSAE) Statements on Standards for Attestation Engagements apply to engagements in which a practitioner is engaged to issue or does issue an examination, a review, or an agreed-upon procedures report on subject matter, or on an assertion about the subject matter, which is the responsibility of another party. An engagement to examine management's assertion that the entity's schedule of investment returns is presented in accordance with specific criteria would fall within this scope. Statements on Standards for Accounting and Review Services apply to engagements involving the unaudited financial statements of a nonissuer. An engagement to examine management's assertion that the entity's schedule of investment returns is presented in accordance with specific criteria does not fall within this scope.

Which type of audit procedures would an auditor use to test a client's financial statement assertions at the account, transaction, or disclosure level? A. Substantive procedure (only) B. Substantive procedures and test of controls C. Analytical procedures (only) D. test of controls (only)

B. Substantive procedures and test of controls -The auditor uses substantive procedures and tests of controls at the relevant assertion level to test a client's material account balances, transaction classes, and disclosure items in the financial statements.

When an auditor tests a computerized accounting system, which of the following is true of the test data approach? A. The program tested is different from the program used throughout the year by the client B. Test data are processed by the client's computer programs under the auditor's control C. Several transactions of each type must be tested D. Test data must consist of all possible valid and invalid conditions

B. Test data are processed by the client's computer programs under the auditor's control Test data consists of "dummy" data run through the client's computer system. The data should be processed under the auditor's control.

When determining the auditor's and management's responsibility for compliance with laws and regulations during an audit, which of the following statements below would be incorrect? A. Management and those charged with governance are responsible for ensuring that the company's operations are conducted in accordance with all applicable laws and regulations B. The auditor is expected to detect the client's noncompliance with all laws and regulations affecting the transaction cycles C. The auditor is not responsible for preventing noncompliance with laws and regulations D. The auditor provides reasonable assurance that the financial statements are free of material misstatement due to noncompliance with laws and regulations

B. The auditor is expected to detect the client's noncompliance with all laws and regulations affecting the transaction cycles This is a false statement. Although an auditor performs preliminary risk assessments and audit procedures on various transaction cycles during an audit, the auditor is not expected to detect the client's noncompliance with all laws and regulations that may apply.

The quarterly data required by SEC Regulation S-K have been omitted. Which of the following statements must be included in the auditor's report? A. The Auditor will review the selected data during the review of the subsequent quarterly financial data B. The company has not presented the selected quarterly financial data C. The company's internal control provides an adequate basis to complete the review D. The auditor was unable to review the data

B. The company has not presented the selected quarterly financial data If the quarterly data required by SEC Regulation S-K have been omitted, the auditor's report must include a statement indicating that the company has not presented such data. Choice "4" is incorrect. The auditor's report should only state that the auditor was unable to review quarterly data required by SEC Regulation S-K when the data have been included, but the auditor has not reviewed such data.

An auditor would express an unmodified opinion with an emphasis-of-matter paragraph added to the auditor's report for: An unjustified accounting change A material weakness in internal control A. Yes No B. Yes Yes C. No Yes D. No No

D. No No An unjustified accounting change may cause the auditor to issue a qualified or adverse opinion. A material weakness must be reported to management and those charged with governance, but would not be disclosed in an emphasis-of-matter paragraph added to an otherwise unmodified opinion.

Which of the following comparisons would an auditor most likely make in evaluating an entity's costs and expenses? A. The current year's accounts receivable with the prior year's account receivable B. The current year's payroll expense with the prior year's payroll expense C. The budgeted current year's sales with the prior year's sales D. The budgeted current year's warranty expense with the current year's contingent liabilities

B. The current year's payroll expense with the prior year's payroll expense The most likely analytical review procedure involving costs and expenses would be to compare the current year's payroll expense (average amount per employee) to the prior year, taking into consideration an average increase in wage rates. This is a very effective technique in auditing payroll expense. Choice "4" is incorrect. The current year's budgeted warranty expense would likely be compared to the current year's actual warranty expense, not to all of the contingent liabilities for the year.

An accountant had begun to audit the financial statements of a nonissuer. Which of the following circumstances most likely would be considered a reasonable basis for agreeing to the entity's request to change the engagement to a compilation? A. the accountant is prevented from examining the minutes of the board of directors' meetings B. The entity's principal creditors no longer require the entity to furnish audited financial statements C. The accountant is prohibited from corresponding with the entity's legal counsel D. The entity's management does not provide the accountant with a signed representation letter

B. The entity's principal creditors no longer require the entity to furnish audited financial statements An audit may be changed to a compilation or review due to a change in client requirements. Since the creditors no longer require audited financial statements, this is a valid reason for requesting a change.

An auditor decides to issue a qualified opinion on an entity's financial statements because a major inadequacy in the entity's electronic accounting records prevents the auditor from applying necessary procedures. The opinion paragraph of the auditor's report should state that the qualification pertains to A. Inadequate disclosure of necessary information B. The possible effects on the financial statements C. A client-imposed scope limitation D. A departure from GAAS

B. The possible effects on the financial statements When an auditor qualifies his opinion because of a scope limitation, the wording in the opinion paragraph should indicate that the qualification pertains to the possible effects on the financial statements and not to the scope limitation itself.

Which of the following factors is most relevant when an auditor considers the client's organizational structure in the context of control risk? A. Management's attitude toward information processing and accounting departments B. The suitability of the client's lines of reporting C. The organization's recruiting and hiring practices D. Physical proximity of the accounting function to upper management

B. The suitability of the client's lines of reporting The suitability of the client's lines of reporting is an important part of the organizational structure, which in turn is a key component of the control environment. Since the control environment has a pervasive effect on the auditor's risk assessment, consideration of the suitability of the client's lines of reporting is quite relevant in evaluating control risk

Which of the following statements is not true of the test data approach to testing an accounting system? A.Only one transaction of each type need be tested B. The test data must consist of all possible valid and invalid conditions C. The test data need consist of only those valid and invalid conditions that interest the auditor D. Test data are processed by the client's computer programs under the auditor's control

B. The test data must consist of all possible valid and invalid conditions While the auditor will frequently use many valid and invalid conditions, it is not feasible to test every possible valid and invalid condition using a test data approach.

Which of the following circumstances most likely would cause an auditor to suspect that there are material misstatements in an entity's financial statements? A. Cash transactions are electronically processed and recorded, leaving no paper audit trail B. There are unusual discrepancies between the entity's records and confirmation replies C. The monthly bank reconciliation ordinarily includes several large outstanding checks D. Management enforces strict budgetary controls over the entity's departmental supervisors

B. There are unusual discrepancies between the entity's records and confirmation replies An auditor most likely would suspect that there are material misstatements in an entity's financial statements if there were unusual discrepancies between the entity's records and confirmation replies. Choice "1" is incorrect. Lack of a paper audit trail for an electronic transaction is unlikely to cause the auditor to suspect that there are material misstatements in an entity's financial statements. The auditor can test those transactions with computer assisted audit techniques.

In determining the sample size for a test of controls, an auditor should consider the likely rate of deviations, the allowable risk of assessing control risk too low, and the : A. Risk of incorrect acceptance B. Tolerable deviation rate C. Nature and cause of deviations D. Population size

B. Tolerable deviation rate The auditor should consider the tolerable rate of deviation from the control structure policies or procedures being tested, the likely rate of deviations, and the allowable risk of assessing control risk too low The risk of incorrect acceptance is relevant for substantive tests of details rather than tests of controls.

Which of the following circumstances most likely would cause an auditor to consider whether material misstatements exist in an entity's financial statements? A. Significant deficiencies in internal control previously communicated to management are not corrected B. Transactions selected for testing are not supported by proper documentation C. Management places little emphasis on meeting earnings projections D. The board of directors makes all major financing decisions

B. Transactions selected for testing are not supported by proper documentation Vouching recorded transactions to supporting documentation provides support for the occurrence assertion. The lack of supporting documentation calls into question the occurrence of the transactions under examination.

Management of a nonissuer believes and the auditor is satisfied that a material loss probably will occur when pending litigation is resolved. Management is unable to make a reasonable estimate of the amount or range of the potential loss, but fully discloses the situation in the notes to the f/s. If management does not make an accrual in the f/s the auditor should express a (an): A. Qualified opinion due to a scope limitation B. Unmodified Opinion C. Qualified opinion due to a material misstatement of the f/s D. Unmodified opinion with an other-matter paragraph

B. Unmodified Opinion If a contingent liability is probable, but not estimable, and it is disclosed in the footnotes, the auditor should issue an unmodified opinion.

Management believes and the auditor is satisfied, that a pending lawsuit is reasonably possible to result in a material loss of $100,000. The lawsuit has been adequately disclosed in the notes to the financial statements; however, no loss amount has been accrued related to the pending lawsuit. If management does not make an accrual in the financial statements, the auditor should express a(n): A. Unmodified opinion with an emphasis-of-matter paragraph B. Unmodified opinion C. Qualified or adverse opinion D. Qualified or disclaimer

B. Unmodified opinion Lawsuits that are "reasonably possible" require disclosure only. Management has adequately disclosed this situation and, therefore, the event is appropriately accounted for under GAAP. The auditor should express an unmodified opinion.

As part of audit procedures performed on a client's transactions, the auditor reconciles supporting schedules to the corresponding general ledger entries. The auditor would most likely be testing which of the following assertions? A. Understandability and classification B. Valuation, allocation, and accuracy C. Cutoff D. Existence and occurrence

B. Valuation, allocation, and accuracy If the auditor performs a reconciliation of supporting schedules to the client's general ledger entries, the auditor would most likely be testing the valuation, allocation, and accuracy assertion for a particular transaction cycle.

An auditor is concerned about a policy of management override as a limitation of internal control. Which of the following tests would best assess the validity of the auditor's concern? A. Tracing sales orders to the revenue account B. Verifying that approved spending limits are not exceeded C. Matching purchase orders to accounts payable D. Reviewing minutes of board meetings

B. Verifying that approved spending limits are not exceeded If spending limits are exceeded, then this would be evidence that the spending limit control was violated and that management overrode internal control.

Which of the following is a true statement regarding documentation requirements for analytical procedures? A. When an analytical procedure is used during the overall review stage of the audit, the auditor is required to document the auditor's expectation and any additional procedures performed to investigate significant unexplained differences B. When an analytical procedure is used as the principal substantive test of a significant financial statement assertion, the auditor is required to document both the auditor's expectation and the factors considered in developing that expectation C. When an analytical procedure is used as the principal substantive test of a significant financial statement assertion, the auditor is required to document his or her expectation and management's concurrence with that expectation D. When an analytical procedure is used as the principal substantive test of a significant financial statement assertion, the auditor is required to document the reasons analytical procedures were performed instead of tests of details

B. When an analytical procedure is used as the principal substantive test of a significant financial statement assertion, the auditor is required to document both the auditor's expectation and the factors considered in developing that expectation When an analytical procedure is used as the principal substantive test of a significant financial statement assertion, the auditor is required to document both the auditor's expectation and the factors considered in developing that expectation. Choice "1" is incorrect. When an analytical procedure is performed during the overall review stage, there are no specific documentation requirements. The requirement that the auditor document the expectation and any additional procedures performed to investigate significant unexplained differences relates to analytical procedures performed as principal substantive tests.

Which of the following events would least likely indicate the existence of related party transactions? A. Making a loan with no scheduled date for the funds to be repaid B. Writing off obsolete inventory to net realizable value just before year end C. Borrowing funds at an interest rate significantly below prevailing market rates D. Maintaining compensating balance arrangements for the benefit of principal stockholders

B. Writing off obsolete inventory to net realizable value just before year end

which of the following factors most likely would heighten an auditor's concern about the risk of fraudulent financial reporting? A. Management's disclosure of unresolved litigation and contingent liabilities B. Yr-end adjustments by the entity that significantly affect financial results C. A lack of competition in the entity's industry, accompanies by increasing profit margins D. The audit committee's approval of the initial selection of accounting principles

B. Yr-end adjustments by the entity that significantly affect financial results Year-end adjustments recorded by the entity that significantly affect financial results would most likely heighten an auditor's concern about the risk of fraudulent financial reporting.

In its annual report to shareholders, Lake Co. included a separate management report that contained additional information. Lake's auditor is expressing an unmodified opinion on Lake's financial statements but has not been engaged to examine and report on this additional information. What is the auditor's responsibility concerning such a report? A. The auditor should add an other-matter paragraph to the report on the financial statements disclaiming an opinion on the additional information B. the auditor should read the management report and consider whether it contains a material misstatement of fact C. The auditor should request Lake to place the management report in its annual report where it will not be misinterpreted to be the auditor's assertion D. The auditor has no obligation to read the management report or to verify the accuracy or appropriateness of its contents

B. the auditor should read the management report and consider whether it contains a material misstatement of fact The auditor should read other information accompanying the basic financial statements and consider whether it contains a material inconsistency or material misstatement of fact

A CPA firm would best provide itself reasonable assurance of meeting its responsibility to offer professional services that conform with professional standards by: A. Establishing an understanding with each client concerning individual responsibilities in a signed engagement letter B.Maintaining a comprehensive system of quality control that is suitably designed in relation to its organizational structure C. Assessing the risk that errors and fraud may cause the financial statements to contain material misstatements D. Developing specific audit objectives to support management's assertions that are embodied in the financial statements

B.Maintaining a comprehensive system of quality control that is suitably designed in relation to its organizational structure Statements of Quality Control Standards (SQCS) are issued to provide guidance with respect to audit quality control. These standards indicate that the nature and extent of a firm's quality control policies and procedures are directly tied to its organizational structure. So by maintaining a comprehensive system of quality control in relation to its organizational structure, the CPA firm would provide reasonable assurance that its professional services conform to the above standards.

At December 31, Yr 2, Curry Co. had the following balances in selected asset accounts: Yr 2. inc. over Yr1 Cash $300 $100 A/R net $1200 $400 Inventory $500 $200 Prepaid exp. $100 $40 Other assets $400 $150 Total assets $2500 $890 Curry also had current liabilities of $1,000 at Dec. 31, Yr 2, and sales (net) of $7,200 for the year then ended. What was Curry's day sales in accounts receivable enduring Year 2 A. 50.7 B. 40.6 C. 60.8 D. 30.4

C. 60.8 The days sales in accounts receivable is calculated by dividing ending accounts receivable (net) by the sales (net) / 365. Using the amounts given, Curry's days sales in accounts receivable during Year 2 is calculated as follows: Days sales in accounts receivable = $1,200 / ($7,200 / 365) = 60.8 days

which of the following does not demonstrate an inappropriate segregation of duties? a. an accounting clerk receives customer payments and records the resulting reduction in accounts receivable B. The cashier performs the monthly bank reconciliation C. A billing clerk prepares invoices and records the resulting increase in accounts receivable D. The purchasing manager approves vendor invoices for payment

C. A billing clerk prepares invoices and records the resulting increase in accounts receivable Preparing invoices and recording the related receivables are both recordkeeping functions that would not be inconsistent with each other. Choice "4" is incorrect. This would allow the purchasing manager to purchase and pay for unauthorized goods with little oversight from others.

Which of the following situations represents a risk factor that relates to misstatements arising from misappropriation of assets? A. An inability to generate cash flow from operations B. A strained relationship between management and the predecessor auditor C. A lack of independent checks D. A high turnover of senior management

C. A lack of independent checks A lack of independent checks is a fraud risk factor that provides an opportunity for misappropriation of assets. Choice "1" is incorrect. An inability to generate cash flow from operations may evidence a threat to the company's financial stability or profitability, providing an incentive for fraudulent financial reporting.

To be effective, analytical procedures in the overall review stage of an audit engagement should be performed by: A. The managing partner who has responsibility for all audit engagements at that practice office B. The staff accountant who performed the substantive auditing procedures C. A manager or partner who has a comprehensive knowledge of the client's business and industry D. The CPA firm's quality control manager or partner who has responsibility for the firm's peer review program

C. A manager or partner who has a comprehensive knowledge of the client's business and industry The manager and partner on a specific job generally bear a great deal of responsibility for the audit and the report. Typically they would perform analytical procedures during the final review stage, to evaluate overall financial statement presentation and to assess the conclusions reached. In order to evaluate the results of the analysis and to perform an effective review, the manager or partner should have a comprehensive knowledge of the client's business and the industry.

In which case would an unmodified opinion not be appropriate? A. There is a change in accounting principle that has a material effect on the current year financial statements B. There is an unjustified departure from GAAP, but it does not have a material effect on the financial statements C. A material related party transaction has occurred and has been accounted for appropriately, but it has not been adequately disclosed in the financial statements D. There is a justified departure from GAAP

C. A material related party transaction has occurred and has been accounted for appropriately, but it has not been adequately disclosed in the financial statements -Inadequate disclosure of a material related party transaction would result in a qualified or adverse opinion. -An immaterial unjustified departure from GAAP would not affect the unmodified opinion. Note that if the effect were material, a qualified or adverse opinion would be appropriate. -A material change in accounting principle would result in the addition of an emphasis-of-matter paragraph to the unmodified opinion.

Which of the following items should be included in an auditor's report for financial statements prepared in conformity with another comprehensive basis of accounting (OCBOA)? A. The signature of the company controller B. A paragraph stating that the audit was conducted in accordance with OCBOA C. A title that includes the word "independent" D. A sentence stating that the auditor is responsible for the financial statements

C. A title that includes the word "independent" -The title of the OCBOA report should be "Independent Auditor's Report." -The financial statements are prepared using the OCBOA. The audit report would state that the audit was conducted in accordance with auditing standards generally accepted in the United States of America.

Analytical procedures performed in the overall review stage of an audit suggest that several accounts have unexpected relationships. The results of these procedures most likely would indicate that: A. Irregularities exist among the relevant account balances B. The communication with those charged with governance should be revised C. Additional tests of details are required D. Internal control activities are not operating effectively

C. Additional tests of details are required If analytical procedures suggest unexpected relationships, the auditor would perform additional tests of details of the accounts involved. Choice "1" is incorrect. The identification of unexpected relationships as a result of analytical procedures does not necessarily mean that irregularities exist in the relevant account balances, although this is a possible explanation.

Which of the following statements is a basic element of the auditor's report under US auditing standards? A. The disclosures provide reasonable assurance that the financial statements are free of material misstatement B. The auditor evaluated the overall internal control C. An audit includes evaluating significant estimates made by management D. The financial statements are consistent with those of the prior period

C. An audit includes evaluating significant estimates made by management Under U.S. auditing standards, the auditor's audit report includes a statement that "An audit includes evaluating...significant estimates made by management..." Choice "1" is incorrect. The audit report does not state that disclosures provide reasonable assurance that the financial statements are free of material misstatement. The correct statement is: "...standards require that we plan and perform the audit to obtain reasonable assurance that the financial statements are free of material misstatement."

Which of the following relatively small misstatements most likely could have a material effect on an entity's financial statements? A. An uncollectible account receivable that was not written off B. A petty cash fund disbursement that was not properly authorized C. An illegal payment to a foreign official that was not recorded D. A piece of obsolete office equipment that was not retired

C. An illegal payment to a foreign official that was not recorded An illegal payment of an otherwise immaterial amount could be material if there is a reasonable possibility that it could lead to a material contingent liability or a material loss of revenue.

which of the following is least likely to aid the auditor in evaluating the risk of improper revenue recognition due to fraud? A. Trend analysis of revenues and sales returns by month B. Comparison of sales volume, as determined from recorded revenue amounts, with production capacity C. Analysis of sales commissions over the most recent 5-yr period D. Comparison of revenue reported by month and by product line fro the current and prior years

C. Analysis of sales commissions over the most recent 5-yr period Items other than revenue may affect sales commissions (e.g., changes to the commission structure), and therefore analyzing sales commissions is least likely to aid the auditor in evaluating the risk of improper revenue recognition due to fraud.

Which of the following is an element of a CPA firm's quality control system that should be considered in establishing its quality control policies and procedures? A. Using statistical sampling techniques B. Complying with laws and regulations C. Assigning personnel to engagements D. Considering audit risk and materiality

C. Assigning personnel to engagements The AICPA's Statements on Quality Control Standards assert that a system of quality control for a firm encompasses the firm's organizational structure and the policies and procedures established by the firm in order to provide reasonable assurance of conforming to professional standards. Toward that end, policies and procedures for human resources, including assigning personnel to engagements, should be established to provide reasonable assurance that the persons assigned will have the technical training and proficiency required to perform their work and progress within the firm

For a particular entity's financial statements to be presented fairly in conformity with the applicable financial reporting framework it is not required that the principles selected: A. Reflect transactions in a manner that presents the financial statements within a range of acceptable limits B. Present information in the financial statements that is classified and summarized in a reasonable manner C. Be applied on a basis consistent with those follows in the prior year D. Be appropriate in the circumstances for the particular entity

C. Be applied on a basis consistent with those follows in the prior year For a particular entity's financial statements to be presented fairly in accordance with the applicable financial reporting framework, it is not required that the principles selected be applied on a basis consistent with those followed in the prior year, merely that any changes in accounting principle be properly accounted for and disclosed. -the principles selected must reflect transactions in a manner that present the FS within a range of acceptable limits.

An auditor most likely would express an unmodified opinion and would not add emphasis-of-matter or other-matter paragraph to the report if the auditor: A. Wishes to emphasize that the entity had significant transactions with related parties B. Concurs with the entity's change in its method of computing depreciation C. Believes that there is a probable likelihood of material loss resulting from an uncertainty that is sufficiently supported and disclosed D. Discovers that supplementary information required by FASB has been omitted

C. Believes that there is a probable likelihood of material loss resulting from an uncertainty that is sufficiently supported and disclosed An auditor most likely would express an unmodified opinion and would not add an additional paragraph to the report if the auditor believes that there is a probable likelihood of a material loss resulting from an uncertainty that is sufficiently supported and disclosed.

Which of the following procedures should a CPA most likely perform in the planning stage of a financial statement audit? A. Make inquiries of the client's attorney regarding pending and threatened litigation and assessments B. Communicate with those charged with governance concerning the prior year's audit adjustments C. Compare recorded financial information with anticipated results from budgets and forecasts D. Obtain representations from management regarding the availability of all financial records

C. Compare recorded financial information with anticipated results from budgets and forecasts The planning process should include application of analytical procedures, such as comparison of the financial statements with budgeted or anticipated results. -Management representations are typically obtained at the end of the audit, not during the planning stage.

Which of the following analytical procedures most likely would be used during the planning stage of an audit? A. Comparing the current-year ratio of aggregate salaries paid to the number of employees to the prior-year's ratio B. Reading the letter from the client's attorney and considering the threat of litigation C. Comparing current-year to prior-year sales volumes D. Reading the financial statements and notes and considering the adequacy of evidence

C. Comparing current-year to prior-year sales volumes The comparison of current year to prior year sales volumes would be a good analytical procedure to perform during the planning stage as it would give the auditor information regarding changes in the client's business during the period. Choice "1" is incorrect. Although this is an analytical procedure, it would likely be one performed during the specific review of salaries expense, as opposed to one performed during the planning stage.

Which of the following procedures would an auditor most likely perform in planning a financial statement audit? A. Searching for unauthorized transactions that may aid in detecting unrecorded liabilities B. Examining computer generated exception reports to verify the effectiveness of internal controls C. Comparing the financial statements to anticipated results D. Inquiring of the client's legal counsel concerning pending litigation

C. Comparing the financial statements to anticipated results A requirement during planning is to perform analytical procedures, which involve comparisons of recorded amounts to expectations.

Which of the following factors would most likely influence the form and extent of the auditor's documentation of an entity's internal control environment? A. Amount of audit work performed by the internal auditor B. Results of verifying material account balances C. Complexity and size of the entity D. Amount of audit work performed at a interim date

C. Complexity and size of the entity The complexity and size of the entity will most likely influence the form and extent of the auditor's documentation of an entity's internal control environment. For example, a smaller, less complex entity will typically have fewer controls than a larger, complex entity and, therefore, most likely would result in less audit documentation about the entity's internal control environment.

PCAOB has established risk assessment standards for issuer audits. Which of the following is not a financial statement assertion as recognized by PCAOB standards? A. Occurrence B. Disclosure C. Cutoff D. Presentation

C. Cutoff Cutoff is not a financial statement assertion recognized by PCAOB standards. PCAOB standards include the assertions of Existence or Occurrence, Completeness, Valuation or Allocation, Rights and Obligations, and Presentation and Disclosure. Cuttoff testing would be covered by the Existence or Occurrence and the Completeness assertions.

An auditor most likely would add an emphasis of matter paragraph when: A. The auditor chooses to report on supplementary information presented with the financial statements in the auditor's report rather than in a separate report B. The financial statements of the prior period were audited by a predecessor auditor and the predecessor's audit report is not reissued C. Describing a justified change in accounting principle that has a material effect on the entity's financial statements D. Restricting the use of the auditor's report

C. Describing a justified change in accounting principle that has a material effect on the entity's financial statements An auditor would add an emphasis-of-matter paragraph when describing a justified change in accounting principle that has a material effect on the entity's financial statements. Choice "1" is incorrect. An auditor would add an other-matter paragraph when the auditor chooses to report on supplementary information presented with the financial statements in the auditor's report, rather than in a separate report.

which of the following statements is correct concerning statistical sampling in tests of controls? A. As the population size increases, the sample size should increase proportionately B. In determining tolerable rate, an auditor considers detection risk and the sample size C. Deviations from specific control activities at a given rate ordinarily result in misstatements at a lower rate D. There is an inverse relationship between the expected population deviation rate and the sample size.

C. Deviations from specific control activities at a given rate ordinarily result in misstatements at a lower rate Deviations from control activities do not necessarily result in misstatements. Therefore, deviations from pertinent control activities at a given rate would ordinarily be expected to result in misstatements at a lower rate. Choice "1" is incorrect. In tests of controls, population size has virtually no effect on sample size unless the population is small

Before applying substantive tests to the details of asset accounts at an interim date, an auditor should assess: A. Inherent risk at a high level B. Materiality for the accounts tested as insignificant C. Difficulty in control the incremental audit risk D. Control risk at a low level

C. Difficulty in control the incremental audit risk Before performing substantive tests at an interim date, the auditor must assess the difficulty in controlling the incremental audit risk from the interim date (on which the substantive procedures are performed) to the year-end date (on which an opinion is rendered).

Restrictions imposed by a client prohibit the observation of physical inventories, which account for 35% of all assets. Alternative audit procedures cannot be applied, although the auditor was able to examine satisfactory evidence for all other items in the financial statements. The auditor should issue a(an): A. Unmodified opinion with an explanation in an emphasis-of-matter paragraph B. "Except for" qualified opinion C. Disclaimer of opinion D. Qualified opinion with a basis for modification paragraph

C. Disclaimer of opinion Restrictions of scope imposed on the audit of such a large (35%) asset would require a disclaimer of opinion

An auditor's report contains the following sentences: We did not audit the financial statements of JK Co., a wholly owned subsidiary, which statements reflect total assets and revenues constituting 17 & 19% of the related consolidated totals. Those statements were audited by other auditors whose report has been furnished to us and our opinion insofar as it relates to the amounts included for JK Co. is based solely on the report of the other auditors These sentences: A. Qualify the opinion B. Disclaim an opinion C. Divide responsibility D. Are an improper form of reporting

C. Divide responsibility

An audit firm has been hired to perform an audit for a new,large public client. In order to gain an understanding of the client and their environment, the audit firm should gather information on the following factors with the exception of: A. Reviewing the client's corporate objectives and strategies that are used to achieve these objectives B. Obtaining an understanding of the client's industry and regulatory environment C. Evaluating the thoroughness of management's financial performance review to determine if business performance is properly measured D. Understanding the client's selection and application of accounting policies and evaluate whether the accounting policies are appropriate for the client's business

C. Evaluating the thoroughness of management's financial performance review to determine if business performance is properly measured The auditor's purpose is to gain an understanding of the client and the operating environment in which the company operates. While the auditor should review management's financial performance review to determine if the client is meeting their objectives, the auditor's purpose is information gathering and not to determine flaws in the performance evaluation process.

When testing the presentation and disclosure assertion for a given client's transaction cycle, the auditor would consider which of the following the least relevant assertion? A. Rights and obligations B. Valuation and accuracy C. Existence D. Completeness

C. Existence Existence is the least relevant assertion when testing a client's presentation and financial statement disclosure for a transaction cycle. The CVRU acronym should be memorized for the presentation and disclosure category. (It should be noted that existence is a relevant assertion for the account balances and transaction and events categories.)

An internal auditor's work would most likely affect the nature, timing, and extent of an independent CPA's auditing procedures when the internal auditor's work relates to assertions about the: A. Valuation of related party transactions B. Valuation of intangible assets C. Existence of fixed asset additions D. Existence of contingencies

C. Existence of fixed asset additions In making judgments about the extent of the effect of the internal auditor's work on the auditor's procedures, the auditor considers the materiality of financial statement amounts, the risk of material misstatement of the assertions related to these financial statement amounts, and the degree of subjectivity involved in the evaluation of the audit evidence gathered in support of the assertions. As the degree of subjectivity increases, the need for the auditor to perform tests of the assertions increases. Testing the existence of fixed asset additions involves very little subjectivity, and thus work performed by the internal auditor may reduce the auditor's testing in this area Choice "4" is incorrect.. Testing the existence of contingencies involves much subjectivity, and should, therefore, be performed by the auditor.

to determine the sample size for a test of controls, an auditor should consider the tolerable deviation rate, the allowable risk of assessing control risk too low, and the: A. Upper deviation rate B. Risk of incorrect acceptance C. Expected Deviation rate D. Risk of incorrect rejection

C. Expected Deviation rate When determining the sample size for a test of controls, the auditor should consider the expected deviation rate (which is the auditor's best estimate of the deviation rate in the population before the sampling plan is executed), the tolerable deviation rate, and the allowable risk of assessing control risk too low. Choice "1" is incorrect. The upper deviation rate is used to evaluate a balance after a sampling plan has been performed, not to determine the sample size for a test of controls.

Which of the following is required before accepting a new audit engagement? I. Making inquiries of the predecessor auditor regarding management integrity II. Making inquiries of the predecessor auditor regarding matters that may affect the conduct of the audit III. Understanding the prospective client's business and the industry in which it operates A. Only II and III B. Only I and III C. I only D. I, II, and III

C. I only Before accepting a new audit engagement, the auditor is required to make inquiries of the predecessor auditor regarding matters that may bear on management integrity. Making inquiries of the predecessor auditor regarding matters that may affect the conduct of the audit and developing an understanding of the prospective client's business and industry are tasks which may be performed after acceptance of the engagement.

Jewel, CPA, audited Infinite Co.'s prior-year financial statements. These statements are presented with those of the current year for comparative purposes without Jewel's auditor's report, which expressed a qualified opinion. In drafting the current year's auditor's report, Crain, CPA, the successor auditor, should: I. Not name Jewel as the predecessor auditor II. Indicate the type of report issued by Jewel III. Indicate the substantive reasons for Jewel's qualification A. I and II only B. I only C. I,II, and III D. II and III only.

C. I,II, and III If the financial statements of a prior period have been audited by a predecessor auditor whose report is not presented, the successor auditor should indicate in an Other-Matter (Explanatory) paragraph of the report 1) that the financial statements of the prior period were audited by another auditor, 2) the date of the previous report, 3) the type of report issued by the predecessor auditor, and if the report was modified, the substantive reasons therefor and 4) the nature of any emphasis-of-matter, other-matter, or explanatory paragraph included in the previous report. The successor auditor may name the predecessor auditor only if the predecessor auditor's practice was acquired by or merged with that of the successor auditor.

During the course of an audit, the auditor performs an assessment of risks at the assertion level and f/s level. Which of the following is an accurate statement regarding either of these identified risks? A. When assessing risk of material misstatement at the assertion level, the auditor should primarily focus on whether the controls in place are capable of preventing any material misstatements from the identified risk. B. Assertion level risk only pertains to risks identified with specific transactions and account balances C. Identified financial statement level risks could also have a potential impact on many of the client's relevant assertions D. Financial statement level risk would include a client improperly disclosing its derivates activity for the reporting period

C. Identified financial statement level risks could also have a potential impact on many of the client's relevant assertions This represents an accurate statement. Although financial statement risks have a direct impact on the financial statements taken as whole, they could also affect many relevant assertions as well. Choice "1" is incorrect. If the auditor assesses risk of material misstatement at the assertion level, he or she would identify the design and implementation of the controls to determine if they are effective in preventing, detecting, and correcting material misstatements that may result from these risks.

While performing procedures in planning an audit, the auditor's comparison of expectations with recorded amounts yield usual and unexpected relationships. The auditor should consider the results of the analytical procedures in which of the following? A. Identifying significant accounts B. Determining planning materiality and acceptable error C. Identifying the risks of material misstatement due to fraud D. Determining which controls to test

C. Identifying the risks of material misstatement due to fraud The auditor should consider the results of analytical procedures performed in the planning stage of the audit in identifying the risks of material misstatement due to fraud. This is one of the primary purposes of performing analytical procedures during the planning stage.

Which of the following statements regarding directional testing is correct? A. In order to test the completeness assertion, an auditor may select a sample of items from the accounting records, and vouch to their corresponding source documentation B. In order to ensure that an expense has not been understated, an auditor may want to test the existence assertion C. In order to ensure that an asset has not been overstated, an auditor may want to vouch to source documentation D. In order to test the existence assertion, an auditor may select a sample of items and trace to their corresponding source documentation

C. In order to ensure that an asset has not been overstated, an auditor may want to vouch to source documentation An auditor would be likely to test for overstatement of an asset by testing the existence assertion. This test would include vouching down to supporting source documentation. Tracing is an activity that would be performed to test the completeness assertion, by tracing up from source documentation to the summary accounting records.

Which of the following procedures would an auditor ordinarily perform during the review of subsequent events? A. Investigate significant deficiencies in internal control previously communicated to the client B. Analyze related party transactions to discover possible irregularities C. Inquire of the client's legal counsel concerning litigation D. Review the cut-off bank statements for the period after the year-end

C. Inquire of the client's legal counsel concerning litigation An auditor would most likely obtain a letter from the entity's legal counsel describing any pending litigation, unasserted claims, or loss contingencies, to obtain evidence that might impact the year-end financial statements.

Which of the following procedures would an auditor most likely perform to obtain evidence about the occurrence of subsequent events? A Comparing the Financial statements being reported on with those of the prior period B. Investigating personnel changes in the accounting department occurring after year-end C. Inquiring as to whether any unusual adjustments were made after year-end D. Confirming a sample of material accounts receivable established after year-end

C. Inquiring as to whether any unusual adjustments were made after year-end An auditor would most likely inquire as to whether any unusual adjustments were made after year-end that would require adjustment to and/or disclosure in the year-end financial statements Choice "1" is incorrect. Comparing the financial statements being reported on with those of the prior period is not a very good source of subsequent event information.

Which of the following procedures would an auditor most likely perform to obtain evidence about the occurrence of subsequent events? A. investigating changes in stockholders' equity occurring after year-end B. Recomputing a sample of large-dollar transactions occurring after year-end for arithmetic accuracy C. Inquiring of the entity's legal counsel concerning litigation, claims, and assessments arising after year-end D. Confirming bank accounts established after year-end

C. Inquiring of the entity's legal counsel concerning litigation, claims, and assessments arising after year-end The auditor would most likely inquire of the entity's legal counsel concerning litigation, claims and assessments arising after year-end in order to obtain evidence about the occurrence of subsequent events. Claims arising after year-end might well impact the year-end financial statements.

Which of the following is not true about the report release date? A. It is often the date on which the report is delivered to the client B. It is the date on which the auditor grants the client permission to use the report C. It is defined as the date after which existing documentation must not be deleted, and additions to the documentation file must be documented as such D. It is used to define the beginning of the retention period

C. It is defined as the date after which existing documentation must not be deleted, and additions to the documentation file must be documented as such The documentation completion date (and not the report release date) is defined as the date after which existing documentation must not be deleted, and additions to the documentation file must be documented as such. -The report release date is used to define the beginning of the retention period.

Tech Company has disclosed an uncertainty due to pending litigation. The auditor's decision to issue a qualified opinion rather than a unmodified opinion most likely would be determined by the: A. Entity's lack of experience with such litigation B. Inability to estimate the amount of loss C. Lack of sufficient evidence D. Lack of insurance coverage for possible losses from such litigation

C. Lack of sufficient evidence Lack of sufficient evidence to support management's assertions would most likely cause an auditor to issue a qualified or disclaimer of opinion.

An auditor issued an audit report that was dual dated for a subsequent event occurring after the original date of the auditor's report but before issuance of the related financial statements. The auditor's responsibility for events occurring subsequent to the original report date was: A. Limited to include only events occurring up to the date of the last subsequent event referenced B. Extended to subsequent events occurring through the later date C. Limited to the specific event referenced D. Extended to include all events occurring since the original report date

C. Limited to the specific event referenced When an auditor issues a report that is dual dated for a subsequent event occurring after the original date of the auditor's report, but before issuance of the related financial statements, the auditor's responsibility for events occurring subsequent to the original report date is limited to the specific event referenced.

Under which of the following circumstances would the expression of an adverse opinion be inappropriate? A. The company issues financial statements that purport to present financial position and results of operations, but it refuses to include the related statement of cash flows B. Management does not provide reasonable justification for a change in accounting principles C. Management refuses to allow the auditor to contact legal counsel D. The financial statements do not adequately disclose litigation that is probable to result in a material loss

C. Management refuses to allow the auditor to contact legal counsel An auditor may express a qualified or disclaimer of opinion when, due to a scope limitation, the auditor is unable to perform all the tests necessary to complete an audit. Management's refusal to permit inquiry of the attorneys generally will result in a disclaimer of opinion or withdrawal from the audit. Choice "1" is incorrect. The statement of cash flows is a requirement of GAAP to be considered a complete set of financial statements. A material misstatement of financial statements, such as the omission of information that is required to be presented, would result in a qualified or adverse opinion.

Under which of the following circumstances would the expression of an adverse opinion be inappropriate? A. Management does not provide reasonable justification for a change in accounting principles B. The company issues financial statements that purport to present financial position and results of operations, but it refuses to include the related statement of cash flows C. Management refuses to allow the auditor to contact legal counsel D. The financial statements do not adequately disclose litigation that is probable to result in a material loss

C. Management refuses to allow the auditor to contact legal counsel An auditor may express a qualified or disclaimer of opinion when, due to a scope limitation, the auditor is unable to perform all the tests necessary to complete an audit. Management's refusal to permit inquiry of the attorneys generally will result in a disclaimer of opinion or withdrawal from the audit. -The statement of cash flows is a requirement of GAAP to be considered a complete set of financial statements. A material misstatement of financial statements, such as the omission of information that is required to be presented, would result in a qualified or adverse opinion.

When disclaiming an opinion due to a client-imposed scope limitation in an audit of a nonissuer, an auditor should indicate in a separate paragraph why the audit did not comply with generally accepted auditing standards. The auditor should also omit the: Auditor's Responsibility paragraph Opinion Paragraph A. No Yes B. Yes No C. Yes Yes D. No No

D. No No When disclaiming an opinion because of scope limitations, the auditor should indicate in a separate paragraph(s) the reasons that the audit did not comply with GAAS. The Auditor's Responsibility paragraph is revised to mention the disclaimer, but is not omitted. The Opinion paragraph is not omitted; however it indicates that no opinion is expressed.

During the annual audit of Ajax Corp., an issuer, Jones, CPA, a continuing auditor, determined that illegal political contributions had been made during each of the past 7 years, including the year under audit. Jones notified the board of directors about the illegal contributions, but they refused to take any action because the amounts involved were immaterial to the financial statements. Jones should reconsider the intended degree of reliance to be placed on the A. Letter of audit inquiry to the client's attorney. B. Prior years' audit plans. C. Management representation letter. D. Preliminary judgment about materiality levels.

C. Management representation letter. The auditor should consider the implications of an act of noncompliance with laws and regulations in relation to other aspects of the audit, particularly the reliability of representations of management.

The scope of an audit is not restricted when an attorney's response to an auditor as a result of a client's letter of audit inquiry limits the response to: A. The attorney's opinion of the entity's historical experience in recent similar litigation B. An evaluation of the likelihood of an unfavorable outcome of the matters disclosed by the entity C. Matters to which the attorney has given substantive attention in the form of legal representation D. The probable outcome of asserted claims and pending or threaten litigation

C. Matters to which the attorney has given substantive attention in the form of legal representation The scope of an audit is not restricted when an attorney's response is limited to matters to which the attorney has given substantive attention in the form of legal representation. The attorney may also limit his or her response to matters that are considered individually or collectively to be material

An advantage of statistical sampling over nonstatistical sampling is that statistical sampling helps an auditor to A. Eliminate the risk of nonsampling errors B. Reduce the level of audit risk and materiality to a relatively low amount C. Measure the sufficiency of the audit evidence obtained D. Minimize the failure to detect errors and fraud

C. Measure the sufficiency of the audit evidence obtained Statistical sampling helps the auditor to measure the sufficiency of the audit evidence because the auditor can quantify the audit risk, thus assisting in limiting it to an acceptable level. Choice "1" is incorrect. Nonsampling errors relate to the improper evaluation of evidence by the auditor and are not dependent on the sampling method used.

The sample size of a test of controls varies inversely with Expected population deviation rate Tolerable Rate A. Yes No B. Yes Yes C. No Yes D. No No

C. No Yes he sample size for a test of controls varies directly with the expected deviation rate and inversely with the tolerable rate. If the auditor expects more errors, he or she would increase sample size; conversely, if the tolerable rate of deviation increases, not as many items need to be selected.

On February 25, a CPA issued an auditor's report expressing an unmodified opinion on financial statements for the year ended January 31. On March 2, the CPA learned that on February 11, the entity incurred a material loss on an uncollectible trade receivable as a result of the deteriorating financial condition of the entity's principal customer that led to the customer's bankruptcy. Management then refused to adjust the financial statements for this subsequent event. The CPA determined that the information is reliable and that there are creditors currently relying on the financial statements. The CPA's next course of action most likely would be to: A. Notify the entity's creditors that the financial statements and the related auditor's report should no longer be relied on. B. Issue revised financial statements and distribute them to each creditor known to be relying on the financial statements. C. Notify each member of the entity's board of directors about management's refusal to adjust the financial statements D. Issue a revised auditor's report and distribute it to each creditor known to be relying on the financial statements

C. Notify each member of the entity's board of directors about management's refusal to adjust the financial statements Since the material loss affects the audit report and there are creditors relying on the financial statements, the client properly should adjust the financial statements. Since they are refusing to do so, the auditor would most likely notify the board of directors of the situation in an attempt to encourage the adjustment.

Which of the following computer-assisted auditing techniques processes client input data on a controlled program under the auditor's control to test controls in the computer system? A. Integrated test facility B. Review of program logic C. Parallel Simulation D. Test data

C. Parallel Simulation Parallel simulation is a technique in which the auditor reprocesses the client's data using the auditor's own software. The auditor then compares his or her results to those obtained by the client. An integrated test facility uses the auditor's input data on the client's system, on-line.

which of the following is an engagement attribute for an audit of an entity that processes most of its financial data in electronic form without any paper documentation A. Increased effort to search fro evidence of management fraud B. Increased emphasis on the completeness assertion C. Performance of audit tests on continuous basis D. Discrete phases of planning, interim, and year-end fieldwork

C. Performance of audit tests on continuous basis Continuous performance of audit tests is required when financial data is processed electronically, without provision of paper documentation, to ensure that controls are operating effectively throughout the period under audit. Choice "2" is incorrect. The completeness assertion must be tested in all audit engagements, regardless of the method used to process financial data or the adequacy of the paper documentation provided.

When an auditor of a nonissuer qualifies an opinion because of the inability to confirm accounts receivable by direct communication with debtors, the wording of the qualified opinion paragraph of the auditor's report should indicate that the qualification pertains to the: A. Departure from GAAS B. Limitation on the auditor's scope C. Possible effects on the f/s D. Lack of sufficient appropriate audit evidence

C. Possible effects on the f/s When an auditor of a nonissuer qualifies his or her opinion because of a scope limitation, such as the inability to confirm accounts receivable, the wording in the opinion paragraph should indicate that the qualification pertains to the possible effects on the financial statements and not to the scope limitation itself.

Which of the following characteristics would not have an impact in determining the scope of an existing client audit? A. The auditor's knowledge gained from prior experience with the client B. Current regulatory and statutory reporting requirements C. Preliminary evaluations of materiality and audit risk D. The effect of information technology on the audit

C. Preliminary evaluations of materiality and audit risk The auditor's preliminary evaluations of materiality and audit risk will impact the focus of the audit, rather than the scope. -focus of the audit team's efforts -Any knowledge gained by the auditor from prior audit experience with the client will impact the scope of the audit. -Any knowledge gained by the auditor from prior audit experience with the client will impact the scope of the audit. -The scope of the audit will take into account the information technology of the client.

In a computerized payroll system environment, an auditor would be least likely to use test data to test controls related to: A. Missing employee numbers B. Time ticket with invalid job numbers C. Proper approval of overtime by supervisors D. Agreement of hours per clock cards with hours on time tickets

C. Proper approval of overtime by supervisors Proper approval of overtime by supervisors is least likely to be used by the auditor in a "test data" test of controls because it is information generally not recorded in the computer.

The financial statements of Henley Co. contain a material departure from GAAP. Henley effectively argues that, due to unusual circumstances, the finanical statements would have been misleading if they were prepared in conformity with GAAP. Based solely on this information, the auditor should: A. Render an unmodified opinion with no additional paragraphs of explanation B. Render a qualified (except for) opinion with an added paragraph preceding the opinion paragraph explaining the reason for the qualification. The paragraph is titled "Basis for Qualified Opinion" C. Render an unmodified opinion with a separate paragraph explaining the unusual situation D. Render a disclaimer of opinion based on lack of sufficient appropriate audit evidence

C. Render an unmodified opinion with a separate paragraph explaining the unusual situation

To obtain an understanding of a continuing client's business in planning an audit, an auditor most likely would: A. Reevaluate the client's internal control environment B. Read specialized industry journals C. Review prior-year audit documentation and the permanent file for the client D. Perform tests of details of transactions and balances

C. Review prior-year audit documentation and the permanent file for the client Knowledge of an entity's business is ordinarily obtained through experience with the entity or its industry and inquiry of personnel of the entity. Audit documentation from prior years may contain useful information about the nature of the business, its organizational structure, its operating characteristics, and transactions that may require special consideration.

If the predecessor auditor refuses to give the current auditor of a nonissuer access to the documentation, what should the current auditor do? A. Disclaim an opinion due to a scope limitation B. Discuss the matter with the client's legal counsel C. Review the risk assessment of the opening balances of the financial statements D. Withdraw from the engagement

C. Review the risk assessment of the opening balances of the financial statements If the predecessor auditor refuses to give the current auditor of a nonissuer access to the documentation, the current auditor should review the risk assessment of the opening balances of the financial statements.

Audit documentation should: A. Not be permitted to serve as a reference source for the client B. Not contain critical comments concerning management C. Show that the accounting records agree or reconcile with the financial statements D. Be considered the primary support for the financial statements being audited

C. Show that the accounting records agree or reconcile with the financial statements Audit documentation is not the support for the financial statement. The client's books and records are the support for the financial statement. Audit documentation should be the principal support for the work the auditor has done to support the opinion provided on the auditor's report

Which of the following would be an appropriate title for a statement of revenue and expenses prepared using an other comprehensive basis of accounting (OCBOA)? A. Statement of operations B. Income Statement C. Statement of income-regulatory basis D. Statement of activites

C. Statement of income-regulatory basis -Other comprehensive basis of accounting financial statements include financial statements prepared in accordance with a regulatory basis of accounting. An income statement prepared in accordance with a regulatory basis of accounting could be entitled "Statement of income-regulatory basis." -A. This is another term for an income statement under U.S. GAAP and is not an appropriate title for a statement of revenue and expenses prepared using an other comprehensive basis of accounting.

Regardless of the assessed level of control risk, an auditor would perform some A. Analytical procedures to verify the design of internal control B. Dual purpose tests to evaluate both the risk of monetary misstatement and preliminary control risk C. Substantive tests to restrict detection risk for significant transaction classes D. Tests of controls to determine the effectiveness of internal control

C. Substantive tests to restrict detection risk for significant transaction classes Regardless of the assessed level of control risk, an auditor would perform some level of substantive tests to restrict detection risk for significant transaction classes. Even with the lowest possible assessed level of control risk, substantive testing cannot be entirely eliminated for significant transaction classes or balances. Choice "1" is incorrect. Analytical procedures are substantive audit procedures used by the auditor to test account balances, not to verify the design of internal controls.

Which of the following matters relating to an entity's operations would an auditor most likely consider as an inherent risk factor in planning an audit? A. The entity's financial data are available only in computer-readable form B. The entity's financial statements are generated at an outside service center C. The entity enters into derivative transactions as hedges D. The entity's fiscal year ends on June 30

C. The entity enters into derivative transactions as hedges The auditor would most likely consider derivative transactions as an inherent risk factor. Derivative transactions entered into as hedges may result in an increased assessment of inherent risk. For example, derivatives entered into as hedges may involve complex calculations and/or may be based on accounting estimates that are subject to significant measurement uncertainty. Choice "1" is incorrect. The auditor would consider the availability of data, such as the financial data is only available in computer-readable form, when evaluating control (not inherent) risk.

which of the following audit procedures most likely will involve sampling? A. Analyses of controls to determine the appropriate segregation of duties B. Tests of automated application controls when effective information technology general controls are present C. Testing of process for approval of credit to customers for sales on account D. Risk assessment procedures performed to obtain an understanding of internal control

C. Testing of process for approval of credit to customers for sales on account An auditor most likely would use sampling when testing the process for approval of credit to customers for sales on account. Audit sampling is the application of an audit procedure to less than 100 percent of the items in the account balance or class of transactions for the purpose of evaluating some characteristic (in this case, proper approval) Choice "4" is incorrect. An auditor generally does not use sampling when performing risk assessment procedures to obtain an understanding of internal control. The primary goal of risk assessment procedures is to gain an understanding of the accounting system or other relevant parts of the internal control, rather than to evaluate a characteristic of all transactions processed.

In which of the following circumstances would an auditor not express an unmodified opinion? A. Quarterly financial data required by the SEC has been omitted B. The auditor wishes to emphasize an unusually important subsequent event C. The auditor is unable to obtain audited financial statements of a consolidated investee D. There has been a material change between periods in accounting principles

C. The auditor is unable to obtain audited financial statements of a consolidated investee -The inability to obtain audited financial statements of a consolidated investee represents a scope limitation that may result in either a qualified opinion or a disclaimer of opinion. -Omission of selected quarterly data required by SEC regulations is disclosed in an other-matter paragraph added to an otherwise unmodified opinion. -A material change in accounting principles between periods is disclosed in an emphasis-of-matter paragraph added to an otherwise unmodified opinion.

Which of the following is a true statement regarding other information included in documents containing audited financial statements? A. The auditor is required to reference the other information in the audit report on the financial statements B. The auditor may choose to add an emphasis-of-matter paragraph, which includes a disclaimer of opinion on other information, int he audit report on the financial statements C. The auditor may choose to add an other matter paragraph which includes a disclaimer of opinion on other information, in the audit report on the financial statements D. The auditor has no responsibility for other information as long as it is outside the basic fanatical statements

C. The auditor may choose to add an other matter paragraph which includes a disclaimer of opinion on other information, in the audit report on the financial statements The auditor may choose to add an other-matter paragraph on other information, which includes a disclaimer of opinion on other information, in the audit report on the financial statements. Disclaimer of opinion means to deny providing an opinion. The other-matter paragraph will include the statement, "we do not express an opinion . . . ."

Which of the following statements indicates that the auditor has gained a sufficient understanding of a client's internal controls related to the sales order process? A. In a statistically valid sample of 100 sales transactions, the auditor found five undiscovered exceptions and concluded that the system was weak B. The auditor compared sales orders processed with processing clerk head count for three years and noted that processed orders significantly declined while clerk head count remained the same C. The auditor noted in a narrative that the documentation for the sales order system showed the printing of a shipment-exception report listing non-invoiced shipments D. The auditor interviewed the company's supervisor of sales clerks and reviewed six shipment-exception reports that were randomly selected and that showed significant unrecorded balances.

C. The auditor noted in a narrative that the documentation for the sales order system showed the printing of a shipment-exception report listing non-invoiced shipments -This statement shows that the auditor has gained a sufficient understanding of a client's internal controls related to the sales order process. A narrative is one way an auditor can describe his or her understanding of internal control.

when testing a sample of an audit client's bank reconciliations during the year under audit, an auditor notices that several immaterial deposits in transit did not clear the bank in a timely manner. The auditor suspects that there may be fraud. Which of the following audit responses is most appropriate in the situation? A. the auditor should report the finding to the appropriate level of management immediately B. The auditor should focus on material misstatements and not consider these immaterial findings or potential errors further C. The auditor should consider the implications for the integrity of management or employees and the possible effect on other aspects of the audit D. Because the findings are not material, the auditor should project the errors to the population and consider the materiality of the projected misstatement in the auditor's concluding procedures

C. The auditor should consider the implications for the integrity of management or employees and the possible effect on other aspects of the audit Choice "4" is incorrect. Even if fraud is immaterial in amount (quantitatively immaterial), it may be qualitatively material and can have effects on other aspects of the audit.

Which of the following statements is correct concerning an auditor's responsibility to report fraud? A. The auditor is required to communicate to those charged with governance all minor fraudulent acts perpetrated by low-level employees, even if the amounts involved are inconsequential B. The disclosure of material management fraud to principal stockholders is required when both senior management and the board of directors fail to acknowledge the fraudulent activities C. The disclosure of fraudulent activities to parties other than the client's senior management and those charged with governance is not ordinarily part of the auditor's responsibility D. Fraudulent activities involving senior management of which the auditor becomes aware should be reported directly to the SEC

C. The disclosure of fraudulent activities to parties other than the client's senior management and those charged with governance is not ordinarily part of the auditor's responsibility The disclosure of fraudulent activities to parties other than the client's senior management and those charged with governance is not ordinarily part of the auditor's responsibility.

which of the following statements is generally correct about the reliability of audit evidence? A. Reliability of audit evidence refers to the amount of corroborative evidence obtained B. Information obtained indirectly from independent outside sources is more persuasive than the auditor's direct personal knowledge obtained through observation and inspection C. The more effective the internal control structure, the more assurance it provides about the reliability of the accounting data and financial statements D. Reliability of audit evidence refers to the audit evidence obtained from outside the entity

C. The more effective the internal control structure, the more assurance it provides about the reliability of the accounting data and financial statements The reliability of accounting data and financial statements is enhanced by a satisfactory system of internal control.

During the current audit engagement , the auditor inspects several lease contracts pertaining to the client's operating lease transactions. This audit procedure would most likely be testing which of the following assertions? A. Cut-off B. Completeness C. Understandability and classification D. Existence and occurrence

C. Understandability and classification By inspecting lease documentation, the auditor is most likely testing the assertion of understandability and classification. For example, the auditor may want to ensure that selected contracts are indeed classified as operating leases and that the treatment is consistent with the client's financial reporting. Choice "2" is incorrect. Inspecting supporting documents would not be used to test the completeness assertion. Instead the auditor may use analytical procedures, trace transactions from source documents to the financial statements, or observe processes/procedures to test the completeness assertion.

Green, CPA, concludes that there is substantial doubt about JKL Co.'s ability to continue as a going concern. If JKL's financial statements adequately disclose its financial difficulties, Green's auditor's report under U.S. auditing standards should: Include an emphasis-of-matter paragraph following the opinion paragraph Specifically use the words "going concern" Specifically use the words "substantial doubt" A. Yes Yes No B. Yes No Yes C. Yes Yes Yes D. No Yes Yes

C. Yes Yes Yes When a CPA concludes that there is substantial doubt about an entity's ability to continue as a going concern and the entity adequately discloses its financial difficulties, an unmodified opinion is appropriate. An emphasis-of-matter paragraph (following the opinion paragraph) should be used to highlight the situation. This paragraph should include the phrases "substantial doubt" and "going concern."

The auditor should obtain sufficient knowledge of the client's information and communication system relevant to financial reporting to understand all of the following except: A. Control activities related to each account balance, transaction class and disclosure component in the financial statements or to every assertion relevant to them B. Classes of transactions int he entity's operations that are significant to the financial statements and how those transactions are processed from initiation to inclusion in the financial statements C. The means the entity uses to communicate roles, responsibilities, and significant matters relating sot financial report D. The financial reporting process, including development of significant accounting estimates and inclusion of appropriate disclosures

Control activities related to each account balance, transaction class and disclosure component in the financial statements or to every assertion relevant to them Ordinarily, audit planning does not require an understanding of the control activities related to each account balance, transaction class, and disclosure component in the FS or to every assertion relevant to them -The auditor is required to understand significant classes of transactions and how they are processed

Jacob, senior accountant at NEP CPA Firm, is auditing the YR 2 f/s for Top Firmware Corp., a nonissuer and he is currently in the planning stage. Top Firmware Corp. sales historically occur evenly throughout the year. Jacob expects a low likelihood of uncorrected and undetected misstatements. NEP CPA firm's materiality guidelines are as follows: -Overall f/s materiality should be based on either total assets or gross annualized revenue, whichever is larger, and should be calculated by taking the appropriate benchmark and multiplying it by either 1.5 % of the benchmark's assets, or 1% of the benchmark is revenue -Performance materiality is calculated by multiplying the overall materiality by either 80% (for low likelihood of uncorrected and undetected misstatements) or 60% (for high likelihood of uncorrected and undetected misstatements) Selected interim financial information from Top Firm ware Corp: For the period 01/01/YR2-06/30/YR2 Revenue $2.1m Gross profit: $600k As of 06/30/YR2: Total assets: $2m Stockholders' equity: 1.25m Based on the information above what is the overall financial statement materiality? A. $30,000 B. $21,000 C. $60,000 D. $42,000

D. $42,000 Overall materiality = Applicable benchmark × Applicable percentage Overall materiality = $4,200,000 (annualized revenue = $2,100,000 × 2) × 0.01 Overall materiality = $42,000

What best describes the documentation completion date? A. 60 days from the report release date, based on PCAOB standards B. 5 years from the report release date, based on auditing standards C. 7 Years from the report release date, based on auditing standards D. 45 days from the report release date, based on PCAOB Standards

D. 45 days from the report release date, based on PCAOB Standards

An auditor is engaged to report on supplemental information that accompanies audited financial statements. The auditor's report on supplemental information should include: A. A statement that the supplemental information is the responsibility of the auditor B. A separate paragraph at the end of the report restricting its use to specified parties C. A statement that any difference in the facts, circumstances, or assumptions may change the report D. A statement that the supplemental information has been subjected to audit procedures performed in conjunction with the audit of the financial statements

D. A statement that the supplemental information has been subjected to audit procedures performed in conjunction with the audit of the financial statements A statement that "any difference in the facts, circumstances, or assumptions may change the report" appears in an accountant's report on the application of the requirements of an applicable financial reporting framework, but does not appear in the auditor's report on supplemental information.

In an audit of financial statements, an auditor's primary consideration regarding an internal control is whether the control A. Enhances management's decision-making processes B. Reflects management's philosophy and operating style C. Provides adequate safeguards over access to assets D. Affects management's financial statement assertions

D. Affects management's financial statement assertions Assessing control risk is the process of evaluating the effectiveness of an entity's internal control in preventing or detecting material misstatements in the financial statements.

In performing a count of negotiable securities, an auditor records the details of the count on a security count worksheet. What other information is usually included on this worksheet? A. A description of the client's procedures that prevent the negotiation of securities by just one person B. An analysis of realized gains & losses from the sale of securities during the year C. An evaluation of the client's internal control concerning physical access to the securities D. An acknowledgment by a client representative that the securities were returned intact

D. An acknowledgment by a client representative that the securities were returned intact After performing a count of negotiable securities, the auditor would generally obtain an acknowledgment from the client that the securities were returned intact. This helps maintain accountability for the securities, and reduces the likelihood of employee misappropriation (e.g., if a client employee were to steal a security and blame the auditor). Choice "1" is incorrect. The auditor generally would not include a description of the client's control procedures on a security count worksheet, although this information might be included elsewhere in the audit documentation.

Which of the following items is explicitly included in an audit report expressing an unmodified opinion? A. The procedures selected depend on management's approval, including the assessment of the risks of any errors resulting from fraud B. We conducted our audit in accordance with generally accepted accounting principles C. We believe that the audit evidence we have obtained is sufficient and appropriate to provide basis for our review of the financial statements D. An audit involves performing procedures to obtain audit evidence about the amounts and disclosures in the financial statements

D. An audit involves performing procedures to obtain audit evidence about the amounts and disclosures in the financial statements

An entity adequately discloses in the footnotes that the entity is a going concern. An auditor decides to issue an unmodified opinion on the financial statements. The auditor's report should refer to going concern in: A. The introductory paragraph B. An other-matter paragraph C. The Management's responsibility paragraph D. An emphasis-of-matter paragraph

D. An emphasis-of-matter paragraph An other-matter paragraph would not be appropriate, because the matter is appropriately disclosed in the financial statements

Joe Smith has been promoted to audit supervisor prior to an upcoming audit of a large existing client. Assuming that Smith is assigned five assistants for the client audit, his supervisory duties may include all of the following, except for: A. Reviewing the work performed by his assistants to determine adequacy and whether the audit objectives were met B. Staying informed with his assistants regarding audit issues or difficulties encountered with the client C. Communicating to his assistants the susceptibility of the client's financial statements to material misstatement due to error or fraud D. Assuming primary responsibility for all phases of the client audit

D. Assuming primary responsibility for all phases of the client audit The engagement partner (not the audit supervisor) has the primary responsibility for the client audit.

An auditor determines that the entity is presenting certain supplementary financial disclosures of pension information that are required by the GASB. Under these circumstances, the auditor should: A. Add an additional paragraph to the auditor's report that refers to the required supplementary information only if there are problems with the information B. State that the audit is not being performed in accordance with GAAS C. Document in the working papers that the required supplementary information is presented, but should not apply any procedures to the information D. Compare the required supplementary information for consistency with the audited financial statements

D. Compare the required supplementary information for consistency with the audited financial statements The auditor should perform certain limited procedures on supplementary information accompanying the financial statements, including evaluating whether the information is consistent with the audited financial statements.

Reports on special purposed frameworks are issued in conjunction with: A. Pro forma financial presentations designed to demonstrate the effects of hypothetical transactions B. Interim financial information reviewed to determine whether material modifications should be made to conform with GAAP C. Feasibility studies presented to illustrate an entity's results of operations D. Compliance with reporting requirements to be filed with a specific regulator agency

D. Compliance with reporting requirements to be filed with a specific regulator agency A special purpose framework is a financial basis of accounting other than GAAP that includes cash basis, tax basis, regulatory basis, and contractual basis. Reporting to comply with required regulatory requirements fits a special purpose framework that deviates from traditional GAAP reporting.

Blue,CPA, has been asked to issue a written report on the application of the requirements of an applicable financial reporting framework to a specific transaction by an entity that is audited by another CPA. Blue may accept this engagement, but should: A. Report the engagement's findings to the entity's audit committee, the continuing CPA, and management B. Notify the entity that the report is for the general use of all interested parties C. Disclaim any opinion that the hypothetical application of the applicable financial reporting framework conforms with GAAP D. Consult with the continuing CPA to obtain information relevant to the transaction

D. Consult with the continuing CPA to obtain information relevant to the transaction When issuing a report on the application of the requirements of an applicable financial reporting framework to a specific transaction, the reporting CPA should consult with the continuing CPA to obtain information relevant to the transaction. If the reporting accountant decides it is unnecessary to consult with the continuing accountant, then he or she must document the reasons for not consulting. Choice "3" is incorrect. There is no disclaimer in the report; however, the CPA does state that the preparers of the financial statements are responsible for proper accounting treatment.

Which of the following would not be a primary function of an audit strategy? A. Provide the scope of the audit B. Provide a preliminary assessment of materiality and tolerable misstatement C. Outline reporting objectives D. Outline the nature, extent and timing of audit procedures

D. Outline the nature, extent and timing of audit procedures Although the audit strategy and audit plan are linked together, the audit plan (not the audit strategy) outlines the nature, extent, and timing of the audit procedures that will be performed by the auditors on an engagement

A CPA concludes that the unaudited financial statements on which the CPA is disclaiming an opinion are not in conformity with generally accepted accounting principles (GAAP) because management has failed to capitalize leases. The CPA suggests appropriate revisions to the financial statements but management refuses to accept the CPA's suggestions. Under these circumstances, the CPA ordinarily would: A. Issue a qualified opinion or adverse opinion depending on the materiality of the departure from GAAP B. Restrict the distribution of the CPA's report to management and the entity's board of directors C. Express limited assurance that no other material modifications should be made to the financial statements D. Describe the nature of the departure from GAAP in the CPA's report and state the effects on the financial statements, if practicable

D. Describe the nature of the departure from GAAP in the CPA's report and state the effects on the financial statements, if practicable If the client refuses to accept the CPA's suggestions, the CPA should add a paragraph modifying the disclaimer to describe the nature and effect of the departure from GAAP.

Which of the following steps should be performed first in applying analytical procedures? A. Determine whether the difference between expectation and recorded amount is reasonable B. Investigate and evaluate significant differences from the expectation C. Compare the client's recorded balance or ratio with the expectation D. Develop an expectation of a balance or ratio by using relationships that are expected to exist

D. Develop an expectation of a balance or ratio by using relationships that are expected to exist Of the steps listed, "develop an expectation of a balance or ratio by using relationships that are expected to exist" should be performed first when applying analytical procedures.

Which of the following is an element of a CPA firm's quality control policies and procedures applicable to the firm's accounting and auditing practice? A. Professional skepticism B. Technology selection C. Information Processing D. Engagement Performance

D. Engagement Performance The 6 elements are: HELPME -Human Resources -Engagement/Client acceptance and continuance -Leadership Responsibilities -Performance of the engagement -Monitoring -Ethical Requirements

An auditor expressed an adverse opinion on the prior year's f/s because of a lack of adequate disclosure. These statements are properly stated in the current year and presented in comparative form with the current year's financial statements. The auditor's updated report on the prior year's financial statements should: A. Make no reference to the type of opinion expressed on the prior year's f/s B. Express an unmodified opinion with an emphasis-of-matter paragraph (preceding the opinion paragraph) added to the report C. Not change D. Express an unmodified opinion with an emphasis-of-matter paragraph (following the opinion paragraph) added to the report

D. Express an unmodified opinion with an emphasis-of-matter paragraph (following the opinion paragraph) added to the report The emphasis-of-matter paragraph (or an other-matter paragraph) should follow the opinion paragraph to explain the situation.

While planning an engagement to issue on the application of the requirements of an applicable financial reporting framework to a specific transaction of a nonissuer, a reporting accountant should obtain an understanding of the: A. Financial expertise of the users of the reporting accountant's report B. Risk appetites of parties to the specific transaction C. Internal control activities related to the specific transaction D. Form and substance of the specific transaction

D. Form and substance of the specific transaction While planning an engagement to issue a report on the application of the requirements of an applicable financial reporting framework to a specific transaction of a nonissuer, a reporting accountant should obtain an understanding of the form and substance of the specific transaction. -A reporting accountant is not required to obtain an understanding of the internal control activities related to the specific transaction.

The senior auditor responsible for coordinating the fieldwork usually schedules a pre-audit conference with the audit team primarily to: A. Establish the need for using the work of specialists and internal auditors B. Provide an opportunity to document staff disagreements regarding technical issues C. Discuss staff suggestions concerning the establishment and maintenance of time budgets D. Give guidance to the staff regarding both technical and personnel aspects of the audit

D. Give guidance to the staff regarding both technical and personnel aspects of the audit -Pre-audit planning meetings are typically held to plan technical and personnel aspects of the audit. Assistants should be informed of their responsibilities and the objectives of the procedures that they are to perform. -Establishing the need for specialists and internal auditors may be discussed, but this is not the primary reason for the meeting.

How does an auditor of a non issuer make the following representations when issuing the unmodified audit opinion on comparative financial statements? Consistent application of accounting principles Examination of evidence on a test basis A. Implicitly Explicitly B. Explicitly Explicitly C. Explicitly Implicitly D. Implicitly Implicitly

D. Implicitly Implicitly Consistency is implicit in the auditor's report, and will be explicitly mentioned in an emphasis-of-a-matter paragraph only if there are issues with consistency. There is no explicit reference to "test basis." The report says "An audit involves performing procedures to obtain audit evidence about the amounts and disclosures in the financial statements."

Which of the following should the auditor of a nonissuer do when reporting on supplementary information that is required by a designated accounting standard setter, presented with the basic financial statements? A. Make no reference to the required supplementary information in the report. B. Include a reference to the required supplementary information in the opinion paragraph. C. Include a paragraph before the opinion paragraph that references the required supplementary information D. Include a paragraph after the opinion paragraph that references the required supplementary information

D. Include a paragraph after the opinion paragraph that references the required supplementary information The audit report for nonissuers should include an other-matter paragraph when reporting on required supplementary information that is presented with the basic financial statements. This paragraph should appear after the opinion paragraph.

which of the following disclaimers of liability included within a response to an auditor's confirmation request would allow the auditor to rely on the confirmation as appropriate audit evidence for an audit of a non-issuer? A. Information is not guaranteed to be accurate or current and could be a matter of opinion B. Information in the confirmation may not be relied upon by the recipient C. Information is obtained from electronic data sources, which might not contain certain information in the respondent's possession D. Information is furnished as a matter of courtesy without a duty to do so and without responsibility, liability or warranty, expressed or implied

D. Information is furnished as a matter of courtesy without a duty to do so and without responsibility, liability or warranty, expressed or implied Restrictions that appear to be boilerplate disclaimers of liability do not affect the reliability of the information being confirmed. Examples of such disclaimers include the following: Information is furnished as a matter of courtesy without a duty to do so and without responsibility, liability, or warranty, expressed or implied. The reply is given solely for the purpose of the audit without any responsibility on the part of the respondent, its employees, or its agents, and it does not relieve the auditor from any other inquiry or the performance of any other duty. Choice "2" is incorrect. This statement casts doubt about the accuracy of the information in the confirmation and makes the confirmation less reliable. In this case, the auditor may consider it appropriate to seek clarification from the respondent.

In assessing the objectivity of internal auditors, the independent CPA who is auditing the entity's financial statements most likely would consider the: A. Materiality of the accounts recently inspected by the internal auditors B. Tests of internal control activities that could detect errors and fraud C. Results of the tests of transactions recently performed by the internal auditors D. Internal auditing standards developed by the institute of internal auditors

D. Internal auditing standards developed by the institute of internal auditors Objectivity is reflected by the organizational level to which the internal auditor reports as well as by policies prohibiting audits of areas where the internal auditor lacks independence. In assessing the objectivity of internal auditors, the independent CPA who is auditing the entity's financial statements considers information obtained from previous experience, from discussions with management, from external quality reviews (if performed), and from professional internal auditing standards (such as those developed by The Institute of Internal Auditors). Choice "3" is incorrect. Results of the tests of transactions recently performed by the internal auditor would aid the auditor in evaluating the internal auditor's competence rather than his or her objectivity.

An auditor should restrict the use of the auditor's communication related to the audit of a nonissuer's financial statements by including an alert when the written communication: A. Discloses material weaknesses in internal control that elevate the risk of misstatements in the financial statements B. Documents a financial risk that the auditor discussed with those charged with governance on previous audit engagement C. Describes a potential change in accounting methods and its potential effect on the results of operations and financial position D. Is based on measurement or disclosure criteria that the auditor determined to be suitable only for a limited number of users who can be presumed to have an adequate understanding of the criteria

D. Is based on measurement or disclosure criteria that the auditor determined to be suitable only for a limited number of users who can be presumed to have an adequate understanding of the criteria An auditor should restrict the use of the auditor's communication related to the audit of a nonissuer's financial statements by including an alert when the written communication is based on measurement or disclosure criteria that the auditor determined to be suitable only for a limited number of users who can be presumed to have an adequate understanding of the criteria.-> criteria when a restricted use paragraph should be included Choice "3" is incorrect. The auditor often will restrict use to management and those charged with governance when a written communication describes a potential change in accounting methods and its potential effect on the results of operations and financial position. However, this is not the best answer choice because this is only one of the many circumstances that the auditor may restrict the use of the communication. The correct answer choice includes the criteria for when a restricted use paragraph should be included.

Which of the following statements most likely represents a disadvantage for an entity that keeps microcomputer-prepared data files rather than manually prepared files? A. Attention is focused on the accuracy of the programming process rather than errors in individual transactions B. It is usually more difficult to compare recorded accountability with physical count of assets C. Random error associated with processing similar transactions in different ways is usually greater D. It is usually easier for unauthorized persons to access and alter the files

D. It is usually easier for unauthorized persons to access and alter the files It is easier to access and alter microcomputer data files than manually prepared data files. Microcomputer access security is difficult to maintain because of the increased number of data entry points and the potential ability to defeat access controls. Choice "2" is incorrect. There is no significant difference between comparing physical counts with accounting records kept either manually or on an automated system.

Which of the following is true about the term "factual misstatement?" A. It includes both identified misstatements and an estimate of unidentified misstatements B. It refers to misstatements that are the auditor's best estimate of misstatements in the population, based on the auditor's projection of the audit sample to the entire population C. It refers to the cumulative effect of misstatements from prior years on the current year's financial statements D. It refers to misstatements about which there is no doubt

D. It refers to misstatements about which there is no doubt Choice "2" is incorrect. "Projected misstatements" are the auditor's best estimate of misstatements in the population, based on the auditor's projection of the audit sample to the entire population.

While working on a current audit for an insurance company, the auditor discovers that the client uses an actuary to assist in technical matters related to the preparation of the company's financial statements. The actuary above would best be described as a(n): A. External accounting expert B. General Specialist C. Auditor's Specialist D. Management specialist

D. Management specialist The actuary would be considered a management specialist as he or she does not provide audit or accounting work but is a specialist employed by the company to assist in technical matters related to the preparation of the financial statements.

What is the most likely course of action that an auditor would take after determining that performing substantive tests on inventory will take less time than performing tests of controls? A. Assess control risk at a low level B. Perform only tests of controls on inventory C. Perform both tests of controls and substantive tests on inventory D. Perform only substantive tests on inventory

D. Perform only substantive tests on inventory . If it would take less time or be more efficient to perform substantive tests than it would to perform tests of controls, and if there is no other reason to test controls (i.e., if there is not a high degree of electronic processing), the auditor would not be likely to test controls.

An auditor's special report on financial statements prepared in conformity with the cash basis of accounting should include an emphasis-of-matter paragraph after the opinion paragraph that: A. Justifies the reasons for departing from GAAP B. States whether the financial statements are fairly presented in conformity with another comprehensive basis of accounting C. Explains how the results of operations differ from financial statements prepared in conformity with GAAP D. Refers to the note to the financial statement that describes the basis of accounting

D. Refers to the note to the financial statement that describes the basis of accounting An auditor's special report on financial statements prepared in conformity with the cash basis of accounting should include an emphasis-of-matter paragraph after the opinion paragraph that refers to the note to the financial statements that describes the basis of accounting.

Which of the following circumstances most likely would cause an auditor to consider whether material misstatements exist in an entity's financial statements? A. Differences are discovered during the client's annual physical inventory count B. Clerical errors are listed on a monthly computer-generated exception report C. Significant deficiencies in internal control previously communicated have not been corrected D. Supporting records that should be readily available are frequently not produced when requested

D. Supporting records that should be readily available are frequently not produced when requested Supporting records that should be readily available but are frequently not produced when requested would cause an auditor to consider whether material misstatements exist. Choice "3" is incorrect. Failure to correct significant deficiencies in internal control may represent a conscious decision by management to accept that degree of risk because of cost or other considerations. Although this is a fraud risk factor, the auditor is more likely to be concerned about missing audit evidence.

A nonissuer requests that a CPA change an audit engagement to a review engagement. If the accountant agrees to the change, how, if at all, should the accountant's review report be modified? A. The accountant should include in the review report a reference to the original engagement but not the reason for the change B. The accountant should include in the review report the circumstances that resulted in the change in engagement C. The accountant should include in the review report a disclaimer of an audit opinion D. The accountant should issue the review report without mentioning the change in engagement

D. The accountant should issue the review report without mentioning the change in engagement If the accountant agrees with the change, the accountant should issue the review report without mentioning the change in engagement.

An auditor is planning to use substantive analytical procedures in the current audit. Which of the following is not a primary factor that affects the efficiency and effectiveness of these procedures in detecting possible misstatements by the client? A. The data used to develop an expectation is available and reliable B. There is a clear understanding of the relationships among data C. A more precise expectation developed will result in greater effectiveness for identifying misstatements D. The analytical procedures should be applied to assertions that have potential misstatements which are apparent form an examination of detailed evidence

D. The analytical procedures should be applied to assertions that have potential misstatements which are apparent form an examination of detailed evidence The use of analytical procedures for testing assertions is most effective and efficient if the potential misstatements are not apparent from an examination of the detailed evidence or if insufficient details are available. Choices "1", "2", and "3" are incorrect. Each represents a primary factor impacting the efficiency and effectiveness of applying analytical procedures to detect potential material misstatements by the client.

The two requirements crucial to achieving audit efficiency and effectiveness with a computer are: A. Client data that can be accessed by the auditor's computer and audit procedures that are generally applicable to several clients in a specific industry B. The appropriate software to perform the selected audit tasks and client data that can be accessed by the auditor's computer C. Audit procedures that are generally applicable to several clients in a specific industry and the appropriate audit tasks for computer applications D. The appropriate audit tasks for computer applications and the appropriate software to perform the selected audit tasks

D. The appropriate audit tasks for computer applications and the appropriate software to perform the selected audit tasks In computer audit applications, efficient and effective system usage requires: Identification of the appropriate audit tasks and Appropriate software to perform the selected audit tasks.

Which of the following statements is not true regarding the auditor's responsibility for subsequent events? A. The auditor has an active responsibility to make continuing inquiries between the date of the financial statements and the date on which sufficient appropriate audit evidence has been obtained B. The auditor has no active responsibility to make continuing inquires after the date of the auditor's report C. The auditor has an active responsibility to make continuing inquires between the date of the financial statements and the date of the auditor's report D. The auditor has an active responsibility to make continuing inquires between the date of the auditor's report and the date on which the report is submitted

D. The auditor has an active responsibility to make continuing inquires between the date of the auditor's report and the date on which the report is submitted The auditor has no active responsibility to make continuing inquiries between the date of the auditor's report and the date on which the report is submitted. The auditor's active responsibility stops on the date of the auditor's report. -The auditor does have an active responsibility to make continuing inquiries between the date of the financial statements and the date of the auditor's report

which of the following represents an inappropriate segregation of duties? A. The AP clerk has access to the purchase order, receiving report, and the vendor invoice B. The accounting clerk responsible for preparing the payroll register also prepares the paychecks C. The accounting manager approves an invoice submitted by an accounting clerk D. The cash receipts clerk credits customer accounts when payment is received

D. The cash receipts clerk credits customer accounts when payment is received the cash receipts clerk should not have both recordkeeping responsibilities and custody of assets Choice "2" is incorrect. Preparing payroll checks and preparing the payroll register are both recordkeeping responsibilities, so there is no violation of the concept of segregation of duties. Remember, until the checks are actually signed, they do not represent assets.

An auditor of a nonissuer should design tests of details to ensure that sufficient audit evidence supports which of the following: A. The effectiveness of internal controls B. The planned level of control risk C. Management's assertions that internal controls exist and are operating efficiently D. The planned level of assurance at the relevant assertion level

D. The planned level of assurance at the relevant assertion level An auditor of a nonissuer should design tests of details to ensure that sufficient audit evidence supports the planned level of assurance at the relevant assertion level.

Which of the following statements is correct concerning an auditor's use of the work of a specialist? A. The reasonableness of the specialist's assumptions and their applications are strictly the auditor's responsibility B. The auditor may not use the work of a specialist in matters material to the fair presentation of the financial statements C. The auditor need not obtain an understanding of the methods and assumptions used by the specialist D. The work of a management specialist who has a contractual relationship with the client may be acceptable under certain circumstances

D. The work of a management specialist who has a contractual relationship with the client may be acceptable under certain circumstances -The purpose of using the work of a specialist is to provide the auditor with specialized skill or knowledge the auditor may lack. The work of a management specialist who has a relationship with a client may be acceptable under certain circumstances. If the management specialist has a relationship with the client, the auditor should assess the risk that the specialist's objectivity might be impaired. If the auditor believes that the relationship might impair the management specialist's objectivity, the auditor should perform additional procedures with respect to the specialist's assumptions, methods, or findings to determine that the findings are not unreasonable or should engage another specialist for that purpose. -Although the appropriateness and reasonableness of methods or assumptions used and their application are the responsibility of the specialist, the auditor should obtain an understanding of the methods or assumptions used in order to determine whether the findings are suitable for corroborating the representations in the financial statements.

An auditor obtains knowledge about a new client's business and its industry in order to A. Make constructive suggestions concerning improvements to the client's internal control B. Develop an attitude of professional skepticism concerning management's financial statement assertions C. Evaluate whether the aggregation of identified misstatements causes the financial statements taken as a whole to be materially misstated D. Understand the events and transactions that may have an effect on the client's financial statements

D. Understand the events and transactions that may have an effect on the client's financial statements The auditor should obtain knowledge of the client's business and its industry in order to determine the effect of transactions, events, and practices on the client's financial statements.

In May, Year 4, an auditor reissues the auditor's report on the Year 2 financial statements at a continuing client's request. The year 2 financial statements are not restated and the auditor does not revise the wording of the report. The auditor should: A. Dual date the reissued report B. Use the current-period auditor's report date on the reissued report C. Use the release date of the reissued report D. Use the original report date on the reissued report

D. Use the original report date on the reissued report If the auditor reissues the audit report at the client's request, the auditor should use the original report date on the reissued report. Use of a subsequent date implies that the auditor has done additional work.

Which of the following factors would most likely influence an auditor's consideration of the reliability of data when performing analytical procedures? A. Whether the data were developed in a computerized or a manual accounting system B. Whether the data were prepared on the cash basis or in conformity with GAAP C. Whether the data were processed in an online system or a batch entry system D. Whether the data were developed under a system with adequate controls

D. Whether the data were developed under a system with adequate controls Strong, effective internal controls improve the reliability of data. Choice "1" is incorrect. The type of accounting system used does not affect the reliability of data. Both computerized and manual accounting systems can provide reliable data, as long as there are appropriate controls in place.

An auditor may issue a qualified opinion under which of the following circumstances? Lack of sufficient appropriate audit evidence Restrictions of the scope of the audit A. Yes, no B. No, no C. No, Yes D. Yes, Yes

D. Yes, Yes An auditor may issue a qualified opinion (or a disclaimer, depending on materiality) when there is a lack of sufficient appropriate audit evidence, or when there are restrictions on the scope of the audit.

Which of the following procedures would an auditor most likely perform to obtain evidence about the occurrence of subsequent events? a. compare the f/s being reported on with those of prior year b. verify inventory pledged under loan agreements by confirming the details with financial institutions c. trace information from shipping docs to sales invoices and sales journal transactions d. inquire about the current status of transactions that were recorded on the basis of preliminary data

D. inquire about the current status of transactions that were recorded on the basis of preliminary data The auditor is likely to inquire about the current status of transactions that were recorded on the basis of preliminary data when obtaining evidence about the occurrence of subsequent events. The additional information may affect the amount and/or disclosures in the financial statements that are under audit.

Which of the following is not a possible reason why a properly designed system of internal control may fail to prevent or detect fraud? A. Inadequate segregation of duties may allow one person to both perpetrate and conceal fraudulent activity B. Management may override controls through its attitudes and actions C. Human error may result in an inappropriate application of controls D. Collusion by two or more individuals may be used to circumvent controls

Inadequate segregation of duties may allow one person to both perpetrate and conceal fraudulent activity Inadequate segregation of duties implies that the system of internal control was not properly designed

If specific information comes to an auditor's attention that implies noncompliance with laws that could result in a material, but indirect effect of the financial statements, the auditor should next a) apply audit procedures specifically directed to ascertaining whether noncompliance has occurred b) seek the advice of an informed expert qualified to practice law as to possible contingent liabilities c) report the matter to an appropriate level of management at least one level above those involved d) discuss the evidence with the client's audit committee, or others with equivalent authority and responsibility

a) apply audit procedures specifically directed to ascertaining whether noncompliance has occurred If specific information comes to an auditor's attention that implies the existence of possible acts of noncompliance with laws and regulations that could have a material, but indirect effect on the financial statements, the auditor should next apply audit procedures specifically directed to ascertaining whether an act of noncompliance with laws and regulations has occurred. The auditor should obtain an understanding of the situation, inquire of management (at a level above those involved), consult legal counsel, and consider applying additional audit procedures if necessary. Choice "1" is incorrect. The auditor would only communicate with those charged with governance regarding detected acts of noncompliance with laws and regulations, not possible acts of noncompliance with laws and regulations . Choice "2" is incorrect. The auditor would inquire of management at a level above those involved after obtaining an understanding of the situation

Hill , CPA, has been retained to audit the financial statements of Monday Co. Monday's predecessor auditor was Post, CPA, who has been notified by Monday that Post's services have been terminated . Under these circumstances, which party should initiate the communications between Hill and Post? a. Hill, the successor auditor. b. Post, the predecessor auditor. c. Monday's controller or CFO. d. The chairman of Monday's board of directors.

a. Hill, the successor auditor. The initiative to communicate with the predecessor auditor rests with the successor auditor. Note, however, that the successor auditor must first receive permission from the client.

After issuing a report, an auditor has no obligation to make continuing inquiries or perform other procedures concerning the audited financial statements, unless: a. Information, which existed at the report date and may affect the report, comes to the auditor's attention. b. Management of the entity requests the auditor to reissue the auditor's report. c. Information about an event that occurred after the date of the auditor's report comes to the auditor's attention. d. Final determinations or resolutions are made of contingencies that had been disclosed in the financial statements.

a. Information, which existed at the report date and may affect the report, comes to the auditor's attention. After issuing a report, an auditor has no obligation to make continuing inquiries or perform other procedures concerning the audited financial statements, unless information, which existed at the report date and may affect the report, comes to the auditor's attention. In this case the auditor would perform procedures to determine if the information affects the report and is important to the external users.

An auditor established a $60,000 tolerable misstatement for an asset with an account balance of $1 ,000,000. The auditor selected a sample of every twentieth item from the population that represented the asset account balance and discovered overstatements of $3,700 and understatements of $200. Under these circumstances, the auditor most likely would conclude that: a. There is an unacceptably high risk that the actual misstatements in the population exceed the tolerable misstatement because the total projected misstatement is more than the tolerable misstatement. b. There is an unacceptably high risk that the tolerable misstatement exceeds the sum of actual overstatements and understatements. c. The asset account is fairly stated because the total projected misstatement is less than the tolerable misstatement. d. The asset account is fairly stated because the tolerable misstatement exceeds the net of projected actual overstatements and understatements.

a. There is an unacceptably high risk that the actual misstatements in the population exceed the tolerable misstatement because the total projected misstatement is more than the tolerable misstatement. Selection of every twentieth item results in a sample that is 5% (1/20 = .05) of the population. The sample results indicate a net overstatement of $3,500 (= $3,700 − $200), which is then projected to the total population as $70,000 (= $3,500 / .05). (Alternatively, 3,500 × 20 = 70,000). Since the projected misstatement of $70,000 exceeds the tolerable misstatement of $60,000, there is an unacceptably high risk that the actual misstatement in the population will exceed the tolerable misstatement.

Holding other planning considerations equal, a decrease in the amount of misstatement in a class of transactions that an auditor could tolerate most likely would cause the auditor to a. Apply the planned substantive tests prior to the balance sheet date. b. Perform the planned auditing procedures closer to the balance sheet date. c. Increase the assessed level of control risk for relevant financial statement assertions. d. Decrease the extent of auditing procedures to be applied to the class of transactions.

b. Perform the planned auditing procedures closer to the balance sheet date. A decrease in the amount of misstatements that the auditor can tolerate will cause the auditor to modify the nature, timing and/or extent of auditing procedures. Performing the planned procedures closer to the balance sheet date is a timing modification that addresses this change.

Under properly designed internal control, the same employee most likely would match vendors' invoices with receiving reports and also: a. Post the detailed accounts payable records. b. Recompute the calculations on vendors' invoices. c. Reconcile the accounts payable ledger. d. Cancel vendors' invoices after payment.

b. Recompute the calculations on vendors' invoices. Matching vendor's invoices with receiving reports provides authorization for payment and is generally performed in the accounts payable department. Recalculation of vendor invoices is compatible with this authorization function. Choice "3" is incorrect. This review procedure (independent verification) should be performed by someone independent of the employee who approved the invoice for payment.

An auditor intends to use the work of an actuary who has a relationship with the client. Under these circumstances, the auditor: a. Is required to disclose the contractual relationship in the auditor's report. b. Should assess the risk that the actuary's objectivity might be impaired. c. Is not permitted to rely on the actuary because of a lack of independence. d. Should communicate this matter to those charged with governance as a significant deficiency in internal control.

b. Should assess the risk that the actuary's objectivity might be impaired. While a specialist who is unrelated to the client will provide the auditor with greater assurance of reliability, a specialist who is related to the client may be acceptable in certain circumstances. In such situations, the auditor would likely perform additional procedures to verify objectivity.

An audit client sells 15 to 20 units of product annually. A large portion of the annual sales occur in the last month of the fiscal year. Annual sales have not materially changed over the past five years. Which of the following approaches would be most effective concerning the timing of audit procedures for revenue? a. The auditor should perform analytical procedures at an interim date and discuss any changes in the level of sales with senior management. b. The auditor should inspect transactions occurring in the last month of the fiscal year and review the related sale contracts to determine that revenue was posted in the proper period. c. The auditor should perform tests of controls at an interim date to obtain audit evidence about the operational effectiveness of internal controls over sales. d. The auditor should review period-end compensation to determine whether bonuses were paid to meet earnings goals.

c. The auditor should perform tests of controls at an interim date to obtain audit evidence about the operational effectiveness of internal controls over sales. Because a large portion of the annual sales occur in the last month of the fiscal year and the number of transactions for the entire year is relatively small, the most effective audit approach would be for the auditor to inspect transactions occurring in the last month of the year and review the sales contracts to determine that revenue was posted in the proper period.


Conjuntos de estudio relacionados

Chapter 23 (Gynecologic Emergencies)

View Set

Health lesson 6:Developing proper eating habits

View Set

Section 4: Georgia Agency Basics Review Exams

View Set